Sunteți pe pagina 1din 76

ISSN 2601-0305

ISSN-L 2601-0305

REVISTA DE MATEMATICĂ
MARINESCU-GHEMECI OCTAVIAN
PUBLICAT, IE ANUALĂ
PENTRU ELEVI S, I PROFESORI

Liceul ,,S, tefan Diaconescu” Potcoava

Anul V, nr. 1/2021

Editura Hoffman
Editori:
Costel BĂLCĂU (redactor s, ef) Costel ANGHEL (redactor)
Florea BADEA (redactor) Mihai Florea DUMITRESCU
(secretar general de redact, ie)
Comitetul s, tiint, ific:
Membri de onoare:
Prof. univ. dr. Victor ALEXANDRU - Universitatea din Bucures, ti
Prof. univ. dr. Tudor BĂLĂNESCU - Universitatea din Pites, ti
Prof. univ. dr. Corneliu UDREA - Universitatea din Pites, ti
Membri:
Costel BĂLCĂU - Universitatea din Pites, ti
Doru CONSTANTIN - Universitatea din Pites, ti
Cezar JOIT, A - I.M.A.R.
Ruxandra MARINESCU-GHEMECI - Universitatea din Bucures, ti
Radu MICULESCU - Universitatea Transilvania din Bras, ov
Cristinel MORTICI - Universitatea Valahia din Târgovis, te
Cristian NICULESCU - Universitatea din Bucures, ti
Comitetul de redact, ie:
Leonard GIUGIUC Marian HAIDUCU Marin IONESCU
Daniel JINGA Marius MÂINEA Marius PERIANU
Florin STĂNESCU Adrian T, URCANU Sorin ULMEANU

Colaboratori:
Delia Ileana BASCH-NAIDIN Eduard BUZDUGAN Luigi Ionut, CATANA
Aurel CHIRIT , Ă Dumitru ILIE Lavinia DUMITRANA
Iuliana ION-IONESCU Oana KRISZTA Cosmin MANEA
Ileana MARINESCU-GHEMECI Iulia MARINESCU-GHEMECI George MIHAI
Adriana MIU Corina Mianda MÎINESCU Constantin MOGOS , ANU
Florin NĂSUI Mariana NĂSUI Grat, iela POPA
Dorin POPA Sorin PELIGRAD Dragos, PETRICĂ
Florina PETRUS , CĂ Valentin RĂDULESCU Cristina SMARANDACHE
Valentin SMARANDACHE Manuela STROE Daniela TACLIT
Marian TELER Nicolae TOMESCU Iuliana TRAS , CĂ

Tehnoredactare computerizată: Mihail TĂNASE, e-mail: mihaimit@yahoo.it


Redact, ia: Liceul ,,S, tefan Diaconescu”, Str. Principală, nr. 197, cod 237355,
Potcoava, tel. 0742123058, e-mail: florin14mihai@yahoo.com
Revista este editată ı̂n colaborare cu Departamentul de Matematică-Informatică,
Universitatea din Pites, ti.
Forma digitală a revistei poate fi accesată la adresa: http://rmgo.upit.ro
Publicată de: Editura Hoffman, www.EdituraHoffman.com, www.LibrariaHoffman.ro
Anul V, Nr. 1, 2021
PROBLEME PENTRU CONCURSURI 3

PROBLEME PENTRU CONCURSURI

Probleme propuse

Clasa a V-a

MGO 161. Într-un centru de ı̂nchiriere de vehicule cu pedale mai sunt disponibile
doar biciclete, triciclete s, i mas, inut, e cu patru rot, i.
Determinat, i câte vehicule sunt din fiecare fel, s, tiind că numărul total de vehicule
este egal cu 20, iar numărul total de rot, i este egal cu 44.
Costel Anghel, Slatina s, i Florea Badea, Scornices, ti

MGO 162. Arătat, i că numărul 2021n poate fi scris ca suma a trei pătrate perfecte
distincte, pentru orice număr natural nenul n.
Marin Chirciu, Pites, ti

MGO 163. Fie A cel mai mic număr natural care are exact 2019 divizori naturali,
iar B cel mai mic număr natural care are exact 2021 de divizori naturali.
Comparat, i numerele A s, i B.
***

MGO 164. Se consideră numărul

N = 123 . . . 9101112 . . . 99100101 . . . 99910001001 . . . 20202021.

a) Arătat, i că numerele N , N + 400 s, i N + 900 nu sunt pătrate perfecte.


b) Există numere naturale k astfel ı̂ncât numărul N + 100k să fie pătrat perfect?

Stelian Corneliu Andronescu s, i Costel Bălcău, Pites, ti

MGO 165. Determinat, i cifrele a, b s, i c astfel ı̂ncât a < b < c s, i suma

S = a,(bc) + b,(ca) + c,(ab)

să fie număr natural.


Costel Anghel, Slatina s, i Florea Badea, Scornices, ti
4 Probleme propuse

Clasa a VI-a

MGO 166. Arătat, i că numărul


A = 73n+2 + 73n+1 + 1
se divide cu 19, pentru orice număr natural n.
Marin Chirciu, Pites, ti
MGO 167. Se consideră numărul
N = 123 . . . 9101112 . . . 99100101 . . . 99910001001 . . . 20202021.

a) Calculat, i restul ı̂mpărt, irii numărului N la 11.


b) Arătat, i că numărul N + 100 nu este pătrat perfect.
Stelian Corneliu Andronescu s, i Costel Bălcău, Pites, ti
MGO 168. Fie ABCD un dreptunghi cu AB = 2021 m s, i BC = 3 m.
Determinat, i cel mai mic număr natural nenul n cu proprietatea că oricum am
alege n puncte distincte pe latura [AB] s, i 2021 de puncte distincte pe latura [CD],
există trei dintre acestea care să formeze un triunghi având aria mai mică sau
egală cu 1 m2 .
***
MGO 169. Fie ABC un triunghi ı̂n care AD este ı̂nălt, ime, AM este mediană,
D, M ∈ (BC), astfel ı̂ncât simetricul punctului D fat, ă de dreapta AM este un
punct E situat pe latura AC s, i m (^DM E) = 120◦ . Fie N punctul de intersect, ie
a dreptelor AM s, i DE. Arătat, i că:
BD BC
a) = ;
DN AC
MN AM
b) = .
DN AC
Costel Anghel, Slatina s, i Florea Badea, Scornices, ti
MGO 170. Se consideră un triunghi ABC astfel ı̂ncât cercurile ı̂nscrise ı̂n triun-
ghiurile ADB s, i ADC sunt tangente, unde AD este bisectoarea unghiului BAC,
D ∈ (BC).
a) Stabilit, i natura triunghiului ABC.
b) Dacă I1 este centrul cercului ı̂nscris ı̂n triunghiul ADB s, i E este punctul
de tangent, ă a laturii AB la acest cerc, aflat, i măsura unghiului ABC astfel ı̂ncât
punctele E, I1 s, i D să fie coliniare.
Costel Anghel, Slatina s, i Florea Badea, Scornices, ti
Probleme propuse 5

Clasa a VII-a

MGO 171. Determinat, i n ∈ N cu proprietatea că numărul

N = 2n · an + 4a + 1

este pătrat perfect pentru orice a ∈ N.

Costel Anghel, Slatina s, i Florea Badea, Scornices, ti

MGO 172. Demonstrat, i că pentru orice n ∈ N∗ ecuat, ia

x4n = y 2n−1 + z 2n+1

are solut, ii ı̂n mult, imea numerelor naturale nenule.

Dorin Mărghidanu, Corabia

MGO 173. Pentru orice număr natural n se consideră numerele reale


√ √
a = n s, i b = n + 23.

a) Există n ∈ N astfel ı̂ncât ı̂ntre a s, i b să fie exact cinci numere naturale?
b) Există n ∈ N astfel ı̂ncât ı̂ntre a s, i b să nu fie niciun număr natural?
În caz afirmativ, determinat, i cel mai mic număr n cu proprietatea dată.

Costel Anghel, Slatina s, i Florea Badea, Scornices, ti

MGO 174. Fie ABCDEF un hexagon regulat având latura AB = a, a > 0 s, i


centrul O. Determinat, i cea mai mică valoare a numărului a pentru care există
o dreaptă d ce intersectează segmentele (AB), (OB), (OC), (OD) s, i (DE) ı̂n
punctele M , N , P , Q, respectiv R astfel ı̂ncât d ∦ AE s, i toate segmentele
M A, M B, N O, N B, P O, P C, QO, QD, RD, RE au lungimile numere naturale.

Mihai Florea Dumitrescu, Potcoava

MGO 175. Fie ABCD un dreptunghi, E un punct pe latura AD, F punctul de


intersect, ie a dreptelor BE s, i CD, iar G punctul de intersect, ie a dreptelor CE s, i
AB. Demonstrat, i că BCF G este un patrulater circumscriptibil dacă s, i numai
dacă BC = 2AB.

Titu Zvonaru, Comănes, ti


6 Probleme propuse

Clasa a VIII-a

MGO 176. Fie a, b, x, y, z ∈ Z. Arătat, i că dacă unul dintre numerele ax2 +by 2 −z 2
s, i bx2 + ay 2 − z 2 este divizibil cu a + b, atunci s, i numărul

ab(x2 + y 2 )2 + (a − b)2 x2 y 2 + z 4

este divizibil cu a + b.
Mihály Bencze, Bras, ov
MGO 177. Determinat, i cea mai mică valoare x ∈ R pentru care inegalitatea
√ √
t2 + 4t t ≥ 6t + 20 t − 4x

are loc pentru orice t ∈ N.


Costel Anghel, Slatina s, i Florea Badea, Scornices, ti
MGO 178. Fie numerele reale a1 , a2 , . . . , an s, i b1 , b2 , . . . , bn , n ∈ N∗ , astfel ı̂ncât

a1 + a2 + . . . + an = 7, b1 + b2 + . . . + bn = 5

s, i ak + bk > 0 pentru orice k ∈ {1, 2, . . . , n}. Demonstrat, i că


a1 b1 a2 b2 an bn
+ + ... + < 3.
a1 + b1 a2 + b2 an + bn
Dorin Mărghidanu, Corabia
MGO 179. Fie a, b s, i c trei numere naturale ce reprezintă lungimile laturilor
unui triunghi neisoscel. Demonstrat, i că:
a) pentru orice k ∈ [0, 1] avem

(a + kb)3 + (b + kc)3 + (c + ka)3 ≤ (k + 1)3 (a + b + c)3 − 3(5 + 6k)(6 + 7k)(7 + 5k);

b) pentru orice k ∈ [1, +∞) avem

(a + kb)3 + (b + kc)3 + (c + ka)3 ≤ (k + 1)3 (a + b + c)3 − 3(5k + 6)(6k + 7)(7k + 5).


Stelian Corneliu Andronescu s, i Costel Bălcău, Pites, ti
MGO 180. Fie V ABCD o piramidă regulată de muchie laterală V A = a, a fiind
un număr real pozitiv fixat. S, tiind că volumul piramidei este maxim, determinat, i:
a) aria bazei;
b) măsura unghiului diedru dintre planele (V AB) s, i (V CD).
***
Probleme propuse 7

Clasa a IX-a

MGO 181. Arătat, i că pentru orice a, b, c ∈ R astfel ı̂ncât ab + bc + ca = 3 avem

4a + 1 4b + 1 4c + 1
+ 2 + 2 ≤ 1.
7a2 + 4a + 4 7b + 4b + 4 7c + 4c + 4
Ardak Mirzakhmedov, Kazahstan

MGO 182. Fie k > 1 s, i a, b, c, d > 0 astfel ı̂ncât ab + bc + ca = d. Arătat, i că


 p √  p √  p √ 
a k 2 − 1 + d b k 2 − 1 + d c k 2 − 1 + d ≤ k 3 (a + b)(b + c)(c + a).

Când are loc egalitatea?


Daniel Jinga, Pites, ti

MGO 183. Fie a, b, c ∈ R, ∆ = b2 − 4ac, astfel ı̂ncât a ≥ 1 s, i


√ √
2b + 1 − ∆ + 4c ≥ 1

(expresiile de sub radicali sunt presupuse mai mari sau egale cu 0).
Demonstrat, i că funct, ia f : R → R, f (x) = ax2 + bx + c satisface relat, ia

f (f (x)) ≥ (f (x))2 , ∀ x ∈ R.
Cristinel Mortici, Viforâta

MGO 184. Pe laturile [AB] s, i [AC] ale triunghiului ABC se consideră punctele
P1 , P2 , . . . , P2021 , respectiv Q1 , Q2 , . . . , Q2021 , distincte două câte două, astfel ı̂ncât

AP1 = P1 P2 = P2 P3 = . . . = P2020 P2021 = P2021 B,


AQ1 = Q1 Q2 = Q2 Q3 = . . . = Q2020 Q2021 = Q2021 C.

Determinat, i câte dintre dreptele Pi Qj , i, j = 1, 2021, trec prin centrul de


greutate G al triunghiului ABC.
Stelian Corneliu Andronescu s, i Costel Bălcău, Pites, ti

MGO 185. Demonstrat, i că ı̂n orice triunghi ABC au loc identităt, ile
X 3 X 2 X 9 X 2 2 X 9 X 4
m2a = · a , m2a m2b = · a b , m4a = · a .
4 16 16
Mihály Bencze, Bras, ov
8 Probleme propuse

Clasa a X-a

X 1
MGO 186. Fie x1 , x2 , . . . , xn > 0, n ∈ N, n ≥ 2 s, i m = . Arătat, i că
xi xj
1≤i<j≤n
r
X 1 4 m3 n(n − 1)
q ≤ .
x 3 + x3 8
1≤i<j≤n i j

Când are loc egalitatea?


Daniel Jinga, Pites, ti

MGO 187. Rezolvat, i ı̂n mult, imea numerelor reale ecuat, ia


r
x x x x x 1
4 + 25 + 36 + 144 + 400 + − x2 = 1.
4
Sorin Ulmeanu, Pites, ti

MGO 188. Determinat, i numerele reale k pentru care inegalitatea

a2 + b2 + c2 + 12k ≥ (k + 1)(ab + bc + ca)

are loc pentru orice numere reale nenegative a, b s, i c astfel ı̂ncât abc ≤ a + b + c + 2.
Vasile Cı̂rtoaje, Ploies, ti s, i Leonard Mihai Giugiuc, Drobeta Turnu Severin

MGO 189. Se consideră ecuat, ia

3x 3x2 3x4 1
x− − 2 − 4 = .
x + 3 x + 3x + 9 x + 3x + 18x2 + 54x + 81
3 602

a) Rezolvat, i ecuat, ia ı̂n mult, imea numerelor reale.


b) Determinat, i o solut, ie complexă nereală a ecuat, iei date.
Mihály Bencze, Bras, ov

MGO 190. Fie ABC un triunghi dreptunghic ı̂n A. Determinat, i valorile naturale
ale tangentei unghiului B pentru care ma , la s, i ha pot fi lungimile laturilor unui
triunghi.
Mihai Florea Dumitrescu, Potcoava
Probleme propuse 9

Clasa a XI-a

 
2 −1 0 0
 2 5 0 0 
MGO 191. Se consideră matricea A = 
 0 0 3 −2  .

0 0 1 6
Calculat, i An , n ∈ N∗ .
***

MGO 192. Arătat, i că pentru orice a, b, c ∈ C, exact unul dintre sistemele
 

 ax + by + cz = 0 
 ax + 2y = 1
2x + 3y + 4z = 0 bx + 3y = 1
 
s, i
 x+y+z =1
 
 cx + 4y = 1
x, y, z ∈ C x, y ∈ C
 

este compatibil.
Mihai Florea Dumitrescu, Potcoava

MGO 193. Fie (xn )n≥1 ⊂ (0, 1] un s, ir descrescător, astfel ı̂ncât pentru orice
 
∗ 1 1
n ∈ N ı̂n intervalul , sunt exact n termeni ai s, irului (xn )n≥1 .
n+1 n

√ 2
Demonstrat, i că lim xn = 0 s, i lim n · xn = .
n→∞ n→∞ 2
Cristinel Mortici, Viforâta

MGO 194. Fie n ∈ N∗ , λ > 0 s, i a1 , a2 , . . . , an ∈ R. Demonstrat, i că


n
X n
X
ak
nλ + ak e ≥ (1 + ln λ) eak .
k=1 k=1
Mihály Bencze, Bras, ov
MGO 195. Fie a1 , a2 , . . . , an numere reale nenegative astfel ı̂ncât

a1 + a2 + . . . + an = n.

Demonstrat, i că
n
2
X 1
n(n − n + 1) ≤ a21 + a22 + . . . + a2n + n2 (n − 1).
ai + n − 1
i=1
Vasile Cı̂rtoaje, Ploies, ti s, i Leonard Mihai Giugiuc, Drobeta Turnu Severin
10 Probleme propuse

Clasa a XII-a

MGO 196. Fie S o parte stabilă a lui N ı̂n raport cu adunarea.


Să se determine S, s, tiind că {4, 5} ⊆ S.
Dorin Mărghidanu, Corabia

MGO 197. Fie (R, ∗) un monoid astfel ı̂ncât


xy
x∗2∗y = + x + y, ∀ x, y ∈ R.
2

a) Calculat, i |2 ∗ 2 ∗{z. . . ∗ 2}.


2021 de 2

b) Calculat, i |1 ∗ 1 ∗{z. . . ∗ 1}.


2021 de 1

Stelian Corneliu Andronescu s, i Costel Bălcău, Pites, ti

MGO 198. Fie m, n ∈ N∗ s, i a > 0. Calculat, i integrala


Z
1
dx, x ∈ (0, ∞).
x (x + a)m
n

Daniel Jinga, Pites, ti

MGO 199. Fie f : [0, 1] →  R o funct


 , ie continuă al cărei grafic admite drept
1
centru de simetrie punctul S , 0 s, i fie n s, i k două numere naturale impare,
2
k ≥ 3. Calculat, i integrala
Z 1p
k
f n (x) dx.
0
Marin Chirciu, Pites, ti

MGO 200. Fie a1 , a2 , . . . , an numere reale pozitive, n ≥ 3, astfel ı̂ncât


n
X ai
= 2.
ai + 1
i=1

Demonstrat, i că

1 1 1 n(n − 2)
+ + ... + + ≥ (n − 1)(n − 2).
a1 a2 an a1 + a2 + . . . + an
Vasile Cı̂rtoaje, Ploies, ti s, i Leonard Mihai Giugiuc, Drobeta Turnu Severin
Rezolvarea problemelor din numărul anterior 11

Rezolvarea problemelor din numărul anterior

Clasa a V-a

MGO 121. Determinat, i numerele naturale abc cu a, b s, i c cifre distincte, astfel


ı̂ncât abc = (2a + 2b + 2c)2 .
Aurel Dobos, an, Lugoj
Solut, ie. Pătratele perfecte pare de trei cifre sunt:

100, 144, 196, 256, 324, 400, 484, 576, 676, 784, 900.

Se elimină cele care nu au cifrele distincte, iar dintre cele rămase, prin verificare,
se obt, ine abc = 324.

MGO 122. Fie s, irul de numere 1, 3, 32 , 33 , . . ..


a) Arătat, i că numărul 22019 nu poate fi scris ca sumă de elemente diferite din
acest s, ir.
b) Dar numărul 22020 ?
Nicolae Stăniloiu, Bocs, a
Solut, ie. Orice sumă de termeni diferit, i ai s, irului dat este de forma M9, M9 + 1,
M9 + 3 sau M9 + 4.
a) Avem 22019 = (23 )673 = (9 − 1)673 = M9 − 1 = M9 + 8, ceea ce rezolvă
problema.
b) Avem 22020 = 2 · 22019 = 2(M9 + 8) = M9 + 16 = M9 + 7, deci nici numărul
22020 nu poate fi scris ca sumă de termeni diferit, i ai s, irului dat.

MGO 123. Arătat, i că oricum s-ar alege nouă numere naturale impare mai mici
decât 50, există trei dintre ele care să aibă acelas, i număr de divizori.
***
Solut, ie. Împărt, im numerele naturale impare mai mici decât 50 ı̂n grupe după
numărul lor de divizori astfel:
- grupa 1: cele cu un singur divizor: 1, deci un număr;
- grupa 2: cele cu doi divizori, adică numerele prime: 3, 5, 7, 11, 13, 17, 19, 23,
29, 31, 37, 41, 43, 47, deci 14 numere;
- grupa 3: cele cu trei divizori: 9, 25, 49, deci 3 numere;
12 Rezolvarea problemelor din numărul anterior

- grupa 4: cele cu patru divizori: 15, 21, 27, 33, 35, 39, deci 6 numere;
- grupa 5: cele cu s, ase divizori: 45 deci un număr.
Conform Principiului cutiei, rezultă că oricum am alege nouă numere naturale
impare mai mici decât 50 există trei dintre ele care să aibă acelas, i număr de
divizori (deoarece ı̂n caz contrar, adică dacă nu am avea trei numere cu acelas, i
număr de divizori, atunci am putea alege doar cel mult câte un număr din grupele
1 s, i 5 s, i cel mult câte două numere din grupele 2, 3 s, i 4, deci ı̂n total doar cel mult
8 numere!).

MGO 124. a) Există două numere naturale prime a căror diferent, ă să fie egală
cu 6 s, i a căror sumă să fie un număr de forma 24k+3 , k ∈ N?
b) Dar cu suma de forma 24k+2 , k ∈ N?
Mihai Florea Dumitrescu, Potcoava
Solut, ie. Fie a s, i b numere naturale prime.
a) Avem a + b = 24k+3 s, i a − b = 6, deci 2a = 24k+3 + 6, de unde a = 24k+2 + 3.
Pentru k = 1 obt, inem solut, ia a = 67, b = 61.
b) Avem a + b = 24k+2 s, i a − b = 6, deci 2a = 24k+2 + 6, de unde a = 24k+1 + 3.
Astfel u(a) = 5 s, i cum a este prim rezultă că a = 5. Dar atunci 5 − b = 6, fals.

MGO 125. a) Arătat, i că numerele 36 + 63 s, i 36 · 63 + 1 sunt divizibile cu 7.


b) Fie n ∈ N. Demonstrat, i că 3n + n3 este divizibil cu 7 dacă s, i numai dacă
3n · n3 + 1 este divizibil cu 7.
Costel Anghel, Slatina s, i Florea Badea, Scornices, ti
Solut, ie. a) 36 + 63 = 729 + 216 = 945 = 7 · 135 s, i 36 · 63 + 1 = 272 (7 − 1)3 + 1 =
(28 − 1)2 (7 − 1)3 + 1 = (M7 + 1)(M7 − 1) + 1 = M7 − 1 + 1 = M7.
b) Evident, dacă n = M7 atunci 3n + n3 s, i 3n · n3 + 1 nu se divid cu 7.
Fie acum n 6= M7. Atunci n3 = M7 ± 1.
Pentrun3 = M7 + 1 avem 3n · n3 + 1 − 3n + n3 = 3n · n3 − 3n − n3 − 1 =
  

3n n3 − 1 − n3 − 1 = n3 − 1 (3n − 1) = M7, deci 3n · n3 + 1 este divizibil cu


7 dacă s, i numai dacă 3n + n3 este divizibil cu 7.
Pentrun3 = M7 −1 avem 3n · n3 + 1 + 3n + n3 = 3n · n3 + 3n + n3 + 1 =
  

3n n3 + 1 + n3 + 1 = n3 + 1 (3n + 1) = M7, deci din nou 3n · n3 + 1 este


divizibil cu 7 dacă s, i numai dacă 3n + n3 este divizibil cu 7.
Rezolvarea problemelor din numărul anterior 13

Clasa a VI-a

MGO 126. Determinat, i numerele naturale n pentru care numărul 2020n − 2019n
este divizibil cu 7.
Ionel Tudor, Călugăreni
Solut, ie. Avem 2020n − 2019n = (7 · 288 + 4)n − (7 · 288 + 3)n = M7 + 4n −
M7 − 3n = M7 + (7 − 3)n − 3n = M7 + (−3)n − 3n . Pentru n par obt, inem
că 2020n − 2019n = M7 + 3n − 3n = M7, iar pentru n impar obt, inem că
2020n − 2019n = M7 − 3n − 3n = M7 − 2 · 3n 6= M7 (deoarece 3n nu se divide cu
7). În concluzie, 2020n − 2019n este divizibil cu 7 dacă s, i numai dacă n este par.

MGO 127. Rezolvat, i ı̂n numere ı̂ntregi ecuat, ia x15 + y 9 = 2020.

Stelian Corneliu Andronescu s, i Costel Bălcău, Pites, ti


Solut, ie. Cum x15 s, i y 9 sunt cuburi perfecte, iar orice cub perfect este de forma
M7, M7 + 1 sau M7 + 6, rezultă că

x15 + y 9 ∈ {M7, M7 + 1, M7 + 2, M7 + 5, M7 + 6} .

Cum 2020 = M7 + 4, rezultă că ecuat, ia dată nu are solut, ii.

MGO 128. Fie a, b, c ∈ Q s, i n ∈ N∗ astfel ı̂ncât a + b 6= 0, b + c 6= 0 s, i


a+b c−a n2 + n + 1
(n + 1)a + b = nc. Arătat, i că + = .
b+c a+b n(n + 1)
Marin Chirciu, Pites, ti
Solut, ie. Înlocuind b = nc − (n + 1)a obt, inem

a+b c−a nc − na c−a n(c − a) c−a


+ = + = +
b+c a+b (n + 1)c − (n + 1)a nc − na (n + 1)(c − a) n(c − a)
n 1 2
n +n+1
= + = .
n+1 n n(n + 1)

MGO 129. Fie triunghiul isoscel ABC cu AB = AC s, i ^A = 20◦ . Se construiesc


semidreptele s1 , s2 , . . . , sm , m ∈ N∗ , fiecare având originea ı̂n punctul B s, i fiind
interioară unghiului ^ABC, astfel ı̂ncât unghiul dintre semidreapta si s, i semi-
dreapta [BC să aibă măsura egală cu i · 5◦ , pentru orice i ∈ {1, 2, . . . , m}. Analog,
se construiesc semidreptele t1 , t2 , . . . , tn , n ∈ N∗ , fiecare având originea ı̂n punctul
C s, i fiind interioară unghiului ^ACB, astfel ı̂ncât unghiul dintre semidreapta tj
s, i semidreapta [CB să aibă măsura egală cu j · 3◦ , pentru orice j ∈ {1, 2, . . . , n}.
14 Rezolvarea problemelor din numărul anterior

Oricare două semidrepte si s, i tj , cu i ∈ {1, 2, . . . , m} s, i j ∈ {1, 2, . . . , n}, se inter-


sectează ı̂ntr-un punct; acest punct ı̂mpreună cu punctele B s, i C determină un
triunghi. Notăm cu T mult, imea tuturor acestor triunghiuri.
a) Determinat, i cele mai mari valori posibile pentru numerele m s, i n.
b) Pentru m s, i n determinate la punctul a), calculat, i probabilitatea ca alegând
la ı̂ntâmplare un triunghi din mult, imea T , acesta să fie isoscel sau dreptunghic.
Costel Anghel, Slatina s, i Florea Badea, Scornices, ti
Solut, ie. Evident, ^B = ^C = 80◦ .
a) Avem m · 5◦ < ^B s, i n · 3◦ < ^C, adică m · 5◦ < 80◦ s, i n · 3◦ < 80◦ , deci
cele mai mari valori posibile pentru numerele m s, i n sunt m = 15 s, i n = 26.
b) Cum i ∈ {1, 2, . . . , 15} s, i j ∈ {1, 2, . . . , 26}, rezultă că mult, imea T cont, ine
15 · 26 = 390 triunghiuri.
Triunghiul determinat de si s, i bj cu BC este isoscel dacă s, i numai dacă 5i = 3j,
adică j = 5k s, i i = 3k, adică (i, j) ∈ {(3, 5), (6, 10), (9, 15), (12, 20), (15, 25)}.
Acelas, i triunghi este dreptunghic dacă s, i numai dacă 5i + 3j = 90, adică j = 5k
s, i i = 18 − 3k, adică (i, j) ∈ {(15, 5), (12, 10), (9, 15), (6, 20), (3, 25)}.
Rezultă că mult, imea T cont, ine 5 triunghiuri isoscele s, i tot 5 triunghiuri
dreptunghice, dintre care doar unul este s, i isoscel s, i dreptunghic. Astfel numărul
de triunghiuri isoscele sau dreptunghice din T este 5 + 5 − 1 = 9.
9 3
Obt, inem că probabilitatea cerută este P = = .
390 130
MGO 130. Se consideră un triunghi ABC cu AB = AC, E mijlocul laturii
BC s, i D un punct oarecare pe segmentul (AE). Fie DM ⊥ AB, M ∈ AB s, i
M N ⊥ AC, N ∈ AE. Demonstrat, i că punctul N este mijlocul segmentului (AD)
dacă s, i numai dacă 4ABC este echilateral.
Mihai Florea Dumitrescu, Potcoava
Solut, ie. Fie DQ ⊥ AC, Q ∈ AC. Din 4AM D ≡ 4AQD (cazul I.U.) rezultă că
AM = AQ. Fie {P } = M N ∩ AC, deci N P ⊥ AC, deci N P k DQ.
Astfel avem echivalent, ele:
N este mijlocul segmentului (AD) ⇔ N P este linie mijlocie ı̂n 4DAQ ⇔
AQ AM
AP = ⇔ AP = ⇔ ^AM P = 30◦ (din 4AP M dreptunghic ı̂n P ) ⇔
2 2
^BAC = ^M AP = 60◦ ⇔ 4ABC este echilateral.
Rezolvarea problemelor din numărul anterior 15

Clasa a VII-a

MGO 131. Determinat, i perechile (x, y) de numere naturale nenule cu proprietatea


că
x2 + y 2 · [7x, y] − 91xy = 0,


unde [7x, y] reprezintă cel mai mic multiplu comun al numerelor 7x s, i y.


Stelian Corneliu Andronescu s, i Costel Bălcău, Pites, ti
7x · y
Solut, ie. Ecuat, ia dată poate fi rescrisă ca x2 + y 2 = 13 · , adică
[7x, y]

x2 + y 2 = 13(7x, y)

(unde (7x, y) reprezintă cel mai mare divizor comun al numerelor 7x s, i y).
2 2
 Cazul 1. y 6= M7. Atunci ecuat, ia devine x +y = 13(x, y). Fie d = (x, y), deci
x = da
, (a, b) = 1 s, i ecuat, ia devine d2 (a2 + b2 ) = 13d, adică d(a2 + b2 ) = 13,
y = db
 
2 2 x=a=2 x=a=3
deci d = 1 s, i a + b = 13, deci avem solut, iile si .
y=b=3 , y=b=2
Cazul 2. y = 7y1 , y1 ∈ N∗ . Atunci ecuat, ia devine x2 + 49y12 = 91(x, y1 ).
Rezultă că x = 7x1 , x1 ∈ N∗ s, i ecuat, ia devine 7(x21 + y12 ) = 13(7x1 , y1 ), de unde
∗ 2 2
rezultă că y1 = 7y 2 , y2 ∈ N , iar ecuat, ia devine x1 + 49y2 = 13(x1 , y2 ). Fie
x1 = da
d = (x1 , y2 ), deci , (a, b) = 1 s, i ecuat, ia devine d2 (a2 + 49b2 ) = 13d,
y2 = db
adică d(a2 + 49b2 ) = 13, care nu are solut, ii ı̂n N∗ .

MGO 132. Demonstrat, i că fract, ia

12019 + 22019 + 32019 + 42019 + 52019 + 62019


12020 + 22020 + 32020 + 42020 + 52020 + 62020
este reductibilă.
Ionel Tudor, Călugăreni
Solut, ie. Avem 62019 = (7 − 1)2019 = M7 − 1, 52019 = (7 − 2)2019 = M7 − 22019 s, i
42019 = (7−3)2019 = M7−32019 , deci 12019 +22019 +32019 +42019 +52019 +62019 = M7.
Pe de altă parte, avem 22020 = 23·673+1 = 2 · 8673 = 2(7 + 1)673 = 2(M7 + 1) =
2
M7+2, 42020 = 22020 = (M7+2)2 = M7+4, 32020 = (7−4)2020 = M7+42020 =
M7+4, 52020 = (7−2)2020 = M7+22020 = M7+2 s, i 62020 = (7−1)2020 = M7+1,
deci 12020 + 22020 + 32020 + 42020 + 52020 + 62020 = 2(1 + M7 + 2 + M7 + 4) = M7.
Rezultă că fract, ia dată se simplifică prin 7, deci este reductibilă.
16 Rezolvarea problemelor din numărul anterior

MGO 133. Determinat, i numerele prime p s, i q, p > 3, astfel ı̂ncât

(p − 3)! + 9
− 4225 = q 8 ,
p−2

unde n! = 1 · 2 · 3 · . . . · n, pentru orice n ∈ N∗ .

Costel Anghel, Slatina s, i Florea Badea, Scornices, ti

Solut, ie. Ecuat, ia dată poate fi rescrisă ca (p − 3)! + 9 = (p − 2) q 8 + 4225 . Evident,




pentru p = 4 sau p = 5 nu avem solut, ii, deci p ≥ 6. Prin urmare (p − 3)! este număr
par, deci (p − 3)! + 9 este impar, deci s, i q 8 + 4225 este impar. Astfel q este par s, i
prim, deci q = 2. Ecuat, ia devine (p − 3)! + 9 = 4481(p − 2), care poate fi rescrisă
ca (p − 3) · (p − 4)! + 9 = 4481(p − 3) + 4481, adică (p − 3) [(p − 4)! − 4481] = 4472,
ecuat, ie care are solut, ia unică p = 11.

MGO 134. Se consideră un triunghi neisoscel ABC. Fie G centrul său de


greutate s, i I centrul cercului ı̂nscris ı̂n acest triunghi. Arătat, i că dacă GI ⊥ BC,
b+c
atunci a = , unde a, b s, i c sunt lungimile laturilor BC, AC s, i respectiv AB.
3
Reciproca este adevărată?
Nicolae Stăniloiu, Bocs, a

Solut, ie. Fie ID ⊥ BC, D ∈ (BC). Atunci DB = p − b s, i DC = p − c, unde


a+b+c
p= . Pe de altă parte, folosind Formula medianei obt, inem că GB 2 =
2
2(a2 + c2 ) − b2 2(a2 + b2 ) − c2
s, i GC 2 = . Astfel avem echivalent, ele:
9 9
2(a2 + c2 ) − b2
GI ⊥ BC ⇔ GD ⊥ BC ⇔ GB 2 − GC 2 = DB 2 − DC 2 ⇔ −
9
2(a2 + b2 ) − c2 3(c2 − b2 )
= (p − b)2 − (p − c)2 ⇔ = (c − b)(2p − b − c) ⇔
9 9
(c − b)(c + b) b+c
= (c − b) · a ⇔ a = .
3 3
Prin urmare atât afirmat, ia din enunt, cât s, i reciproca ei sunt adevărate.

MGO 135. Fie ABCD un patrulater cu AB = BC = CD, m (^ABC) = 70◦ s, i


m (^BCD) = 170◦ .
a) Calculat, i măsura unghiului dintre diagonalele patrulaterului.
b) Calculat, i măsurile unghiurilor BAD s, i ADC ale patrulaterului.

Costel Anghel, Slatina s, i Florea Badea, Scornices, ti


Solut, ie. a) Fie O punctul de intersect, ie dintre diagonalele AC s, i BD. În 4ABC
180◦ − 70◦
isoscel avem m (^ACB) = m (^BAC) = = 55◦ , iar ı̂n 4BCD isoscel
2
Rezolvarea problemelor din numărul anterior 17

180◦ − 170◦
avem m (^CBD) = m (^BDC) = = 5◦ . Cum ^AOB este unghi
2
exterior triunghiului BOC, avem m (^AOB) = m (^ACB) + m (^CBD) = 60◦ .
b) Fie punctul E ı̂n interiorul unghiului ABC astfel ı̂ncât m (^ABE) = 60◦ s, i
BE = AB. Atunci 4ABE este echilateral. Cum m (^CBE) = 70◦ − 60◦ = 10◦ ,
rezultă că m (^CBE) + m (^BCD) = 180◦ , deci BE k CD. Dar BE = AB =
BC = CD, deci BCDE este romb. Rezultă că m (^BED) = m (^BCD) = 170◦
s, i ED = EB = EA. Deci E este centrul cercului circumscris triunghiului ABD,
1
^BED este unghi la centru, prin urmare m (^BAD) = · m (^BED) = 85◦ .
2
Astfel m (^ADC) = 360◦ − (85◦ + 70◦ + 170◦ ) = 35◦ .

Clasa a VIII-a

MGO 136. Pentru orice număr natural nenul n considerăm mult, imea
(r r r r )
1 2 3 n
An = , , ,..., .
2020 2020 2020 2020

a) Câte numere n ∈ N∗ au proprietatea că An cont, ine exact 2020 de numere


rat, ionale?
b) Dar exact 2020 de numere irat, ionale?
Stelian Corneliu Andronescu s, i Costel Bălcău, Pites, ti
Solut, ie. Fie n ∈ N∗ s, i k ∈ {1, 2, . . . , n}. Avem echivalent, ele:
2, m ∈ N

r r  2  k = 505mr
k k k = 505m , m ∈ N 
= ∈Q⇔ 2 ≤n ⇔ n
2020 2
2 · 5 · 101 1 ≤ 505m  1≤m≤
505
(unde [x] reprezintă partea ı̂ntreagă a numărului
r  real x), deci numărul de elemente
n
rat, ionale ale mult, imii An este egal cu .
505
r 
∗ n
a) Numărul cerut este numărul de solut, ii din N ale ecuat, iei = 2020.
r 505
n
Aceasta este echivalentă cu 2020 ≤ < 2021, adică cu 505 · 20202 ≤ n <
505
505 · 20212 , deci are 505 · 20212 − 505 · 20202 = 505 · 4041 = 2040705 solut, ii.
r 
∗ n
b) Numărul cerut este numărul de solut, ii din N ale ecuat, iei =
505
n − 2020. Notând n − 2020 = a ∈ N, aceasta este echivalentă, succesiv, cu
18 Rezolvarea problemelor din numărul anterior
"r # r
a + 2020 a + 2020 a + 2020
= a; a ≤ < a + 1; a2 ≤ < a2 + 2a + 1;
505 505 505
505a2 − a − 2020 ≤ 0

. Prima ecuat, ie a acestui sistem are solut, iile naturale
505a2 + 1009a − 1515 > 0
a ∈ {0, 1, 2}, dintre care doar a = 2 verifică s, i a doua ecuat, ie a sistemului. Astfel
a = 2, deci există un singur număr n cu proprietatea din enunt, , s, i anume n = 2022.
1
MGO 137. Pentru orice n ∈ N∗ considerăm numerele an = s, i
n(n + 1)

1 1 1 1
Sn = + + + ... + .
a1 a1 + a2 a1 + a2 + a3 a1 + a2 + a3 + . . . + an

Arătat, i că 68 < S64 < 71.

Costel Anghel, Slatina s, i Florea Badea, Scornices, ti


n n
 
P 1 P 1 1
Solut, ie. Avem a1 + a2 + a3 + . . . + an = = − =
k=1 k(k + 1)  k=1  k k+1
1 n Pn 1 Pn k+1 n
P 1 n 1
P
1− = , deci Sn = = = 1+ = n+ .
n+1 n+1 k=1 k k=1 k k=1 k k=1 k
k+1
Prin urmare
1 1 1 1 1 1
S64 = 64 + 1 + + + ... + = 65 + + + . . . + .
2 3 64 2 3 64
     
1 1 1 1 1 1 1
Astfel avem S64 = 65 + + + + + ... + + + ... + +
   2 3 4 5 8 9 16
1 1 1 1 1 1 1 1 1 1
+ ... + + + ... + > 65+ +2· +4· +8· +16· +32· =
17 32 33 64 2 4 8 16 32 64
1 1 1 1 1 1
65 + + + + + + = 65 + 3 = 68.
2 2 2 2 2 2
     
1 1 1 1 1 1
Pe de altă parte, avem S64 = 65+ + + + ... + + + ... + +
   2 3 4 7 8 15
1 1 1 1 1 1 1 1 1
+ ... + + + ... + + < 65 + 2 · + 4 · + 8 · + 16 · +
16 31 32 63 64 2 4 8 16
1 1 1 1
32 · + = 65 + 5 + = 70 + < 71.
32 64 64 64
MGO 138. Arătat, i că pentru orice x1 ≥ 1, x2 ≥ 2 s, i x3 ≥ 3 astfel ı̂ncât
1 2 3
+ + = s, s ∈ (0, 3), are loc inegalitatea
x1 x2 x3
p √ √ √
(3 − s)(x1 + x2 + x3 ) ≥ x1 − 1 + x2 − 2 + x3 − 3.

Dorin Mărghidanu, Corabia


Rezolvarea problemelor din numărul anterior 19

1 2 3
Solut, ie. Egalitatea dată poate fi scrisă ca 1 − +1− +1− = 3 − s, adică
x1 x2 x3
x1 − 1 x2 − 2 x3 − 3
+ + = 3 − s, deci folosind Inegalitatea Cauchy-Buniakowski-
x1 x2 x3  
x1 − 1 x2 − 2 x3 − 3
Schwarz avem (3−s)(x1 +x2 +x3 ) = (x1 +x2 +x3 ) + + ≥
x1 x2 x3
2
√ √
 r r r
√ x1 − 1 √ x2 − 2 √ x3 − 3
x1 · + x2 · + x3 · = x1 − 1 + x2 − 2 +
x x2 x3
√ 2 1
x3 − 3 , de unde rezultă inegalitatea din enunt, .

MGO 139. Se consideră piramida hexagonală regulată V ABCDEF cu muchia


bazei AB = a. Calculat, i ı̂nălt, imea acestei piramide (ı̂n funct, ie de a), s, tiind că
aria sa laterală este egală cu aria totală a piramidei V ACE.

Costel Anghel, Slatina

Solut, ie. Fie O centrul bazei ABCDEF , M mijlocul lui [AB], N mijlocul lui [AC].
Notăm h = V O.
Pentru piramida V ABCDEF avem
√ r
a 3 p 3a2
OM = , V M = h2 + OM 2 = h2 + ,
2 4
deci aria sa laterală este
r
3a2
Al = 3a · V M = 3a h2 + .
4

Pentru piramida V ACE avem


r
√ a p a2
AC = a 3, ON = , V N = h2 + ON 2 = h2 + ,
2 4
deci aria sa totală este

2· 3

2 3
√ r
AC 3 · AC · V N 3a 3a 3 a2
A0 t = + = + · h2 + .
4 2 4 2 4

Egalitatea Al = A0 t devine, succesiv:


r √ √ r
2
3a2 3a2 3 3a 3 a2
3a h + = + · h2 + ;
4 4 2 4
√ √ √ √ √
2 √4h2 + 3a2 = a 3+ 3· 4h2 + a2 ; 16h2 +12a2 = 3a2 +6a 4h2 + a2 +3(4h2 +a2 );
3a 4h2 + a2 = 2h2 + 3a2 ; 9a2 (4h√ 2 + a2 ) = 4h4 + 12a2 h2 + 9a4 ; 24a2 h2 = 4h4 ;

h2 = 6a2 . Astfel obt, inem că h = a 6.


20 Rezolvarea problemelor din numărul anterior

MGO 140. Se consideră piramida hexagonală regulată V ABCDEF cu muchia


bazei AB = 9 cm s, i ı̂nălt, imea V O = 12 cm. Dintre toate prismele hexagonale
regulate ı̂nscrise ı̂n piramida dată, se consideră aceea de volum maxim. Calculat, i:
a) raportul dintre volumul acestei prisme s, i volumul piramidei;
b) ı̂nălt, imea prismei;
c) distant, a de la V la vârfurile prismei situate pe fet, ele laterale ale piramidei.
Florea Badea, Scornices, ti
Solut, ie. Notăm h1 = ı̂nălt, imea prismei, a1 = aria bazei prismei, v1 = volumul
prismei, h2 = ı̂nălt, imea piramidei, a2 = aria bazei piramidei, v2 = volumul
piramidei. Avem
√ √
3 · AB 2 · 3 243 3 a2 · h2 √
h2 = V O = 12 cm, a2 = = cm2 , v2 = = 486 3 cm3 .
2 2 3
Folosind raportul de asemănare dintre piramida mică, cu vârful V s, i având ca
bază baza superioară a prismei, s, i piramida mare V ABCDEF , avem

a1 (h2 − h1 )2
= ,
a2 h22

27 3
de unde rezultă că a1 = · (12 − h1 )2 s, i prin urmare
32

27 3
v1 = a1 h1 = · h1 · (12 − h1 )2 .
32
Putem rescrie această expresie astfel:

27 3 12 − h1 12 − h1
v1 = · h1 · · .
8 2 2
Folosind Inegalitatea mediilor avem
 3
12 − h1 12 − h1
h1 + +
12 − h1 12 − h1 2 2
h1 · · ≤ = 43 ,
2 2 27
12 − h1
cu egalitate dacă s, i numai dacă = h1 , adică h1 = 4.
2
Rezultă că v1 este maxim pentru h1 = 4 cm.
√ v1 4
Atunci avem v1 = 216 3 cm3 , deci = .
v2 9
VM h2 − h1 VM 8
Fie M ∈ (V A) un vârf al prismei. Avem = , adică = ,
VA h2 15 12
deci V M = 10 cm.
Rezolvarea problemelor din numărul anterior 21

Clasa a IX-a

MGO 141. Rezolvat, i ı̂n mult, imea numerelor reale următorul sistem de ecuat, ii:
(
a2 + b2 + c2 = 1
√ √ .
|a + 4b − 5c| + |b + 4c − 5a| + |c + 4a − 5b| + 14(a + b + c)2 = 3 14
Do Xuan Trong, Vietnam
Solut, ie (Leonard Mihai Giugiuc, Drobeta Turnu Severin). Fie a + b + c = s. Avem
√ √ 1
|s| ≤ 3, iar |s| = 3 dacă s, i numai dacă a = b = c = ± √ .
3
Notăm a + 4b − 5c = x, b + 4c − 5a = y, c + 4a − 5b = z. Atunci x + y + z = 0 s, i
x2 +y 2 +z 2 = −2 (xy + yz + zx) = 2 |xy + yz + zx|. Avem (|x| + |y| + |z|)2 = x2+
y 2 + z 2 + 2 (|xy| + |yz| + |zx|) ≥ x2 + y 2 + z 2 + 2 |xy + yz + zx| = 2 x2 + y 2 + z 2 ,
cu egalitate dacă s, i numai dacă |xy| + |yz| + |zx| = |xy + yz + zx|. Cum x2 + y 2 +
z 2 = 42 a2 + b2 + c2 − 42(ab + bc + ca), deducem că

p
|a + 4b − 5c| + |b + 4c − 5a| + |c + 4a − 5b| ≥ 42 (3 − s2 ).
p √ √ p
Vom demonstra că 42 (3 − s2 ) + 14 · s2 ≥ 3 14, adică 3 (3 − s2 ) + s2 ≥ 3,
√  √ √
adică 3 − s2 3 − 3 − s2 ≥ 0, adevărat, cu egalitate dacă s, i numai dacă

|s| = 3 sau s = 0.
√ 1
De mai sus, |s| = 3 dacă s, i numai dacă a = b = c = ± √ . Să remarcăm că
3
1
a = b = c = ± √ sunt solut, ii ale sistemului.
3
Vom studia ı̂n continuare existent, a solut, iilor ı̂n cazul s = 0. De mai sus,
solut, ii pot exista dacă |xy| + |yz| + |zx| = |xy + yz + zx|. Dar |xy| + |yz| + |zx| =
|xy + yz + zx| dacă s, i numai dacă x · xyz ≥ 0, y · xyz ≥ 0 s, i z · xyz ≥ 0. Dacă,
prin absurd, xyz 6= 0 atunci, cum x + y + z = 0, două numere au semnul opus celui
de-al treilea, ceea ce contrazice x · xyz  ≥20, y 2· xyz 2≥ 0 s, i z · xyz ≥ 0. În concluzie,
 a +b +c =1 3
xyz = 0. Fie cazul x = 0. Avem a+b+c=0 , iar de aici a = ± √ ,

a + 4b − 5c = 0 14
2 1
b = ∓ √ , c = ∓ √ . Cazurile y = 0 s, i z = 0 se tratează analog.
14 14
În concluzie, solut, iile sistemului sunt:
a = b = c = ± √13



 a = ± √3 , b = ∓ √2 , c = ∓ √1

14 14 14 .

 a = ± √114 , b = ∓ √314 , c = ± √214
a = ± √214 , b = ± √114 , c = ∓ √314


22 Rezolvarea problemelor din numărul anterior

MGO 142. Fie a ∈ N∗ . Rezolvat, i ı̂n mult, imea numerelor reale ecuat, ia
   
x x x
− = .
a a+1 2a + 1
Marin Chirciu, Pites, ti
   
x x x x x
Solut, ie. Evident ∈ Z, iar ecuat, ia se rescrie − − + =
2a + 1 a a a+1 a+1
(a2 − a − 1)x
   
x x x
, adică = − . Cum {y} ∈ [0, 1) pentru orice
2a + 1 a(a + 1)(2a + 1) a+1 a
(a2 − a − 1)x
y ∈ R, rezultă că ∈ (−1, 1). Astfel
a(a + 1)(2a + 1)

a2 + a

x x
2a + 1 |a2 − a − 1| , cu 2a + 1 ∈ Z.
<

x x
Cazul 1. Pentru a = 1 obt, inem < 2 cu ∈ Z, deci x ∈ {−3, 0, 3}, iar prin

3 3
verificare rezultă că solut, iile ecuat, iei date sunt doar x ∈ {−3, 0}.
x x
Cazul 2. Pentru a = 2 obt, inem < 6 cu ∈ Z, deci x ∈ {−25, −20, −15, −10,

5 5
−5, 0, 5, 10, 15, 20, 25}, iar prin verificare rezultă că solut, iile ecuat, iei date sunt doar
x ∈ {−15, −5, 0, 5, 10, 20}.
x 12 x
Cazul 3. Pentru a = 3 obt, inem < cu ∈ Z, deci x ∈ {−14, −7, 0, 7, 14},

7 5 7
iar prin verificare rezultă că solut, iile ecuat, iei date sunt doar x ∈ {−7, 0, 7}.
2

x
Cazul 4. Pentru a ≥ 4 obt, inem < a + a < 2, cu x
∈ Z,
2a + 1 a2 − a − 1 2a + 1
deci x ∈ {−2a − 1, 0, 2a + 1}, care verifică ecuat, ia dată.
MGO 143. Fie a, b, c, d numere reale pozitive astfel ı̂ncât ab+bc+cd+da = 4abcd.
Demonstrat, i că
1 1 1 1 √
√ +√ +√ +√ ≤ 2 2.
a3+b3 3
b +c3 3
c +d 3 3
d +a3

Daniel Jinga, Pites, ti


Solut, ie (Daniel Văcaru, Pites, ti). Folosind inegalitatea x3 + y 3 ≥ xy (x + y) pentru
orice x, y > 0 s, i Inegalitatea Cauchy-Buniakowski-Schwarz avem
1 1 1 1
√ +√ +√ +√
a3
+b 3 3
b +c 3 3
c +d 3 d + a3
3
1 1 1 1
≤√ √ +√ √ +√ √ +√ √
ab · a + b bc · b + c cd · c + d da · d + a
r r
1 1 1 1 1 1 1 1
≤ + + + · + + + , (1) .
ab bc cd da a+b b+c c+d d+a
Rezolvarea problemelor din numărul anterior 23
r r
1 1 1 1 ab + bc + cd + da
Conform ipotezei, avem + + + = = 2, (2) .
ab bc cd da abcd
Folosind Inegalitatea mediilor s, i Inegalitatea Cauchy-Buniakowski-Schwarz avem
 
1 1 1 1 1 1 1 1 1
+ + + ≤ √ +√ +√ +√
a+b b+c c+d d+a 2 ab bc cd da
r
1 p 1 1 1 1
≤ · 12 + 1 2 + 1 2 + 1 2 · + + + = 2,
2 ab bc cd da


r
1 1 1 1
deci + + + ≤ 2, (3) .
a+b b+c c+d d+a
Din (1), (2) s, i (3) se obt, ine inegalitatea din enunt, .

MGO 144. Demonstrat, i că ı̂n orice triunghi ABC are loc identitatea
 
A B C A B C
(p − a) sin + (p − b) sin + (p − c) sin = r cos + cos + cos .
2 2 2 2 2 2
Mihály Bencze, Bras, ov
r
A (p − a)(p − b)
Solut, ie (Daniel Văcaru, Pites, ti). Avem (p − a) sin = (p−a)· =
p r 2 bc
p (p − a) (p − b) (p − c) p(p − a) S A A
· = · cos = r cos . Procedând analog
p bc p 2 2
pentru b s, i c s, i adunând relat, iile, obt, inem egalitatea din enunt, .

MGO 145. În triunghiul ABC se ı̂nscrie un semicerc care are centrul pe latura
BC s, i este tangent la laturile AB s, i AC. Se consideră punctele Y ∈ [AC] s, i
Z ∈ [AB] astfel ı̂ncât dreapta Y Z să fie tangentă la semicerc. Fie CY = y s, i
BZ = z. Demonstrat, i că

(b + c)2 yz = b2 − c2 (bz − cy) + a2 bc.




Francisco Javier Garcı́a Capitán, Spania

Solut, ie (Leonard Mihai Giugiuc, Drobeta Turnu Severin). Notăm Y Z = x. Fie


L centrul semicercului dat, deci L este centrul cercului A-exı̂nscris al 4AY Z.
−→ −→
−→ (b − y) AZ + (c − z) AY −→
Cum AZ = c − z s, i AY = b − y, avem AL = . Dar AZ =
b + c − (x + y + z)
−−→ −→ −−→ −→
(c − z) AB −→ (b − y) AC −→ (b − y) (c − z) bAB + cAC
s, i AY = , deci AL = · .
c b bc b + c − (x + y + z)
−→ −−→ −→
Pe de altă parte, B, L, C sunt coliniare, deci AL = v AB + wAC, cu v + w = 1.
Astfel (b − y) (c − z) (b + c) = bc [b + c − (x + y + z)], deci

b2 z + c2 y − yz (b + c) z (b − y) y (c − z)
x= = + .
bc c b
24 Rezolvarea problemelor din numărul anterior

Aplicăm Teorema cosinusului ı̂n 4AY Z s, i obt, inem, succesiv:


(b − y)(c − z)(b2 + c2 − a2 ) z (b − y) y (c − z) 2
 
2 2
(b − y) + (c − z) − = + ;
bc c b
(b − y) (c − z) b2 + c2 − a2 + 2yz

z2 y2
   
(b − y)2 1 − 2 + (c − z)2 1 − 2 = ;
c b bc
(b − y)(c + z) (c − z)(b + y) b2 + c2 − a2 + 2yz
 
(b − y) (c − z) + − = 0;
c2 b2 bc
b2 (b − y)(c + z) + c2 (c − z)(b + y) − bc(b2 + c2 − a2 + 2yz) = 0;
bc(b2 + c2 − b2 − c2 + a2 ) + (bz − cy)(b2 − c2 ) − yz(b2 + c2 + 2bc) = 0;
b2 − c2 (bz − cy) + a2 bc = (b + c)2 yz.


Notă. Domnul Mihai Florea Dumitrescu din Potcoava a propus o altă solut, ie
ı̂n articolul Trei aplicat, ii ale Teoremei cosinusului din revista MATINF nr. 7
(2021).

Clasa a X-a

MGO 146. Demonstrat, i că pentru orice n ∈ N∗ are loc inegalitatea


n
X 1 n
n+1 < .
(k 2 + k) +n (n + 1)2
k=1
Mihály Bencze, Bras, ov
Solut, ia 1. Pentru orice n ∈ N∗
s, i x ≥ 0, conform Inegalităt, ii mediilor avem

n+1
xn+1 + n = xn+1 + 1| + 1 + {z. . . + 1} ≥ (n + 1) xn+1 = (n + 1)x,
n termeni
n+1
cu egalitate dacă s, i numai dacă x = 1. Prin urmare k 2 + k +n > (n+1)(k 2 +k),
pentru orice k = 1, n, deci
n n n
X 1 X 1 1 X 1 n
< = = .
k=1
(k 2 + k)n+1 + n k=1
2
(n + 1)(k + k) n+1 k(k + 1) (n + 1)2
k=1
n+1
Solut, ia 2 (Daniel Văcaru, Pites, ti). Pentru orice k = 1, n avem k 2 + k ≥ 2n+1 ,
1 1 Pn 1 n
deci n+1 ≤ n+1
. Rezultă că n+1 ≤ ,
2
(k + k) +n n + 2 2
k=1 (k + k) +n n + 2n+1
deci este suficient să probăm că n + 2n+1 > (n + 1)2 , adică 2n+1 > n2 + n + 1,
pentru orice n ∈ N∗ . Această inegalitate este adevărată pentru n = 1 s, i n = 2 s, i
se demonstrează prin induct, ie pentru n ≥ 2: dacă 2k+1 > k 2 + k + 1, k ≥ 2, atunci
2k+1 > 2k 2 + 2k + 2 ≥ k 2 + 3k + 3 ≥ (k + 1)2 + (k + 1) + 1.
Rezolvarea problemelor din numărul anterior 25

MGO 147. Rezolvat, i ı̂n mult, imea numerelor reale ecuat, ia


3 √
q p3
13x − 2x − 3x − 3 · 6x−1 − 21x + 2x + 3x + 3 · 6x−1 − 21x − 1
3 √
q p3
= 13x + 3 · 6x−1 − 21x − 1 − 3 · 6x−1 + 21x.
Sorin Ulmeanu, Pites, ti
p
3

Solut
√ , ie (Daniel Văcaru, Pites, ti). Fie √
a= 13x − 2x − 3x − 3 · 6x−1 − 21x, b =
3 x x
√ 2 + 3 +x−1 3·6 x−1 3
− 21x − 1 s, i c = 3 · 6 x−1 + 21x. Rezultă că a3 + b3 + c3 =
x
13 + 3 · 6 − 21x − 1, deci ecuat, ia din enunt, devine (a + b + c)3 = a3 + b3 + c3 ,
adică 3 (a + b) (b + c) (c + a) = 0.
√ x
Cazul 1. a + b = 0, adică a3 + b3 = 0, adică 13 = 42x + 1. Această ecuat, ie
are doar solut, iile x = 0 s, i x = 4, deoarece funct, ia din membrul stâng este strict
convexă, iar funct, ia din membrul drept este liniară.
Cazul 2. b + c = 0, adică b3 + c3 = 0, adică 2x + 3x + 6x = 1. Această ecuat, ie are
solut, ia unică x = −1, deoarece funct, ia din membrul stâng este strict crescătoare.
3 + a3 = 0, adică

Cazul 3. c + a = 0,
x  adică c
x 13x = 2x + 3x . Această ecuat, ie se
2 3
rescrie ca √ + √ = 1, deci are solut, ia unică x = 2, deoarece funct, ia
13 13
din membrul stâng este strict descrescătoare.
În concluzie, mult, imea solut, iilor ecuat, iei date este {−1, 0, 2, 4}.
MGO 148. Fie z1 , z2 s, i z3 numere complexe ce satisfac relat, iile
1 1 1
1 + z1 z2 z3 = 0 s, i z1 + z2 + z3 = + + .
z1 z2 z3
Demonstrat, i că produsul a două dintre numerele z1 , z2 , z3 este egal cu 1.
Dorin Mărghidanu, Corabia
Solut, ia 1. Notând s = z1 + z2 + z3 s, i folosind ipoteza avem
1 1 1 z 1 z2 + z2 z 3 + z 3 z1
s= + + = = −(z1 z2 + z2 z3 + z3 z1 ).
z1 z2 z 3 z1 z2 z 3
Considerând polinomul f (z) = (z − z1 )(z − z2 )(z − z3 ) = z 3 − (z1 + z2 + z3 )z 2 +
(z1 z2 + z2 z3 + z3 z1 )z − z1 z2 z3 = z 3 − sz 2 − sz + 1, avem f (−1) = 0, deci z1 = −1
sau z2 = −1 sau z3 = −1. Cum z1 z2 z3 = −1, rezultă că z2 z3 = 1 sau z1 z3 = 1 sau
z1 z2 = 1.
Solut, ia 2 (Daniel Văcaru, Pites, ti). Cum z1 z2 z3 = −1, rezultă că z1 , z2 , z3 6= 0.
Folosind relat, iile din ipoteză avem (z1 z2 − 1) (z2 z3 − 1) (z3 z1 − 1) = (z1 z2 z3 )2 −
z1 z2 z3 (z1 + z2 + z3 ) + z1 z2 + z2 z3 + z3 z1 − 1 = −z1 z
2 z3 (z1 + z2 + z3 ) + z1 z2 +
1 1 1
z2 z3 + z3 z1 = z1 z2 z3 − (z1 + z2 + z3 ) + + + = 0, de unde se obt, ine
z1 z2 z3
concluzia dorită.
26 Rezolvarea problemelor din numărul anterior

MGO 149. Fie s, irurile (an )n≥1 s, i (bn )n≥1 definite prin

π 3π (2n − 1)π
an = sin + sin + . . . + sin ,
2n + 1 2n + 1 2n + 1
π 3π (2n − 1)π
bn = cos + cos + . . . + cos ,
2n + 1 2n + 1 2n + 1

pentru orice n ∈ N∗ .
a) Studiat, i monotonia celor două s, iruri.
b) Arătat, i că an > bn , pentru orice n ∈ N∗ .

Mihai Florea Dumitrescu, Potcoava


π
Solut, ie. Fie t = s, i z = cos t + i sin t. Avem bn + ian = z + z 3 + . . . + z 2n−1 =
2n + 1
z(z 2n − 1) −1 − z 1 1 cos t − 1 − i sin t
2
= 2 = − = − = − =
z −1
 z − 1 z−1 cos t − 1 + i sin t 2 − 2 cos t
1 t 1 1 t 1 π
1 + ictg , deci bn = s, i an = ctg = ctg .
2 2 2 2 2 2 4n + 2
a) Evident, s, irul (bn )n≥1 este constant. Pentru orice n, m ∈ N∗ cu n < m avem
π π π π  π π π
> s, i , ∈ 0, , deci ctg < ctg , deci
4n + 2 4m + 2 4n + 2 4m + 2 2 4n + 2 4m + 2
an < am . Rezultă că s, irul (an )n≥1 este strict crescător.

1 π 3 1
b) Pentru orice n ∈ N∗ avem an ≥ a1 = ctg = > = bn .
2 6 2 2
MGO 150. Fie d1 s, i d2 două drepte perpendiculare care se intersectează ı̂n punctul
O s, i fie ω1 , ω2 , ω3 s, i ω4 patru cercuri care trec prin O, astfel ı̂ncât ω1 s, i ω2 sunt
tangente la d1 iar ω3 s, i ω4 sunt tangente la d2 .
Demonstrat, i că, indiferent cum s-ar nota celelalte patru puncte de intersect, ie
ale cercurilor date cu X, Y , Z s, i T , avem

OX OZ XZ
· = .
OY OT YT
Cristinel Mortici, Viforâta

Solut, ie. Fie I inversiunea de pol O s, i putere k, cu k > 0 arbitrar fixat. Fie
I(ωj ) = ∆j , j = 1, 4, s, i fie I(X) = X 0 , I(Y ) = Y 0 , I(Z) = Z 0 , I(T ) = T 0 . Atunci
∆1 s, i ∆2 sunt drepte paralele cu d1 , iar ∆3 s, i ∆4 sunt drepte paralele cu d2 , deci
punctele X 0 , Y 0 , Z 0 s, i T 0 sunt vârfurile unui dreptunghi. Avem X 0 Z 0 = Y 0 T 0
XZ YT
(laturi opuse sau diagonale ı̂n dreptunghi), adică k · =k· , deci
OX · OZ OY · OT
OX OZ XZ
· = .
OY OT YT
Rezolvarea problemelor din numărul anterior 27

Clasa a XI-a

MGO 151. Fie A, B, C ∈ M2 (R) astfel ı̂ncât AB + BC + CA = BA + CB + AC.


Demonstrat, i că
 3
det A2 + B 2 + C 2 − AB − BC − CA ≥ [det(A − C) − det(B − C)]2 .
4
Daniel Jinga, Pites, ti
3 2
Solut, ie. Fie ε ∈ C \ R a.ı̂. ε = 1, deci ε + ε + 1 = 0. Folosind s, i ipoteza, avem
(A+εB +ε2 C)(A+ε2 B +εC) = A2 +B 2 +C 2 +ε2 (AB +BC +CA)+ε(BA+CB +
AC) = A2 + B 2 + C 2 + (ε2 + ε)(AB + BC + CA) = A2 + B 2 + C 2 − AB −  BC − CA.
2 2 2
Trecând la determinant, i avem det A + B + C − AB − BC − CA = det(A +
εB + ε2 C) · det(A + ε2 B + εC) = det(A + εB + (−1 − ε)C) · det(A + ε2 B +
(−ε2 − 1)C) = det(A − C − ε(C − B)) · det(A − C − ε2 (C − B)) = f (ε) · f (ε2 ),
unde f (x) = det(A − C − x(C − B)) = x2 det(C − B) + ax + det(A − C), unde
a ∈ R. Fie b = det(C − B) = det(B − C) ∈ R s, i c = det(A − C) ∈ R. Atunci
f (ε)·f (ε2 ) = (bε2 +aε+c)(bε+aε2 +c) = a2 +b2 +c2 +(ε+ε2 )(ab+bc+ca) = a2 +b2 +
b + c 2 3(c − b)2 3(c − b)2
 
2 2
c −ab−bc−ca = a −(b+c)a+b −bc+c = a −2 2 + ≥
2 4 4
s, i astfel rezultă inegalitatea din enunt, .
MGO 152. Fie A, B ∈ Mn (C), unde n ∈ N, n ≥ 2, astfel ı̂ncât A2 = On ,
rang (A) = 1 s, i B 2 = AB 6= On .
a) Demonstrat, i că n > 2 s, i BA = On .
b) Arătat, i că pentru orice n > 2 există o infinitate de matrice cu proprietăt, ile
date.
Florin Stănescu, Găes, ti
Solut, ie. a) Din B = AB rezultă AB = A B = On . Tot din B 2 = AB
2 2 2

rezultă B 3 = AB 2 = On . Dacă n = 2, din B 3 = O2 rezultă det B = 0, deci


B 2 = tr (B) · B, deci tr (B) · B 2 = B 3 = O2 , de unde B 2 = O2 sau tr (B) = 0, care
implică B 2 = tr (B) · B = O2 , deci B 2 = On , contradict, ie. Prin urmare n > 2.
Mai departe, relat, ia B 2 = AB implică B 3 = BAB, deci BAB = On . Din Inega-
litatea lui Frobenius avem rang (BA) + rang (AB) ≤ rang (A) + rang (BAB) = 1,
deci rang (BA) = 0 sau rang (AB) = 0. Cum AB 6= On , obt, inem că BA = On .
b) Pentru orice t ∈ C∗ , matricele de ordin n ≥ 3
   
0 1 0 ... 0 0 1 0 ... 0
 0 0 0 ... 0    0 0 0 ... t 
A= si B = 
... ...  ,

 ... ... ...  ... ... ... ... ... 
0 0 0 .... 0 0 0 0 .... 0
satisfac relat, iile A2 = On , rang (A) = 1 s, i B 2 = AB 6= On .
28 Rezolvarea problemelor din numărul anterior

MGO 153. Fie s, irul (xn )n≥0 definit prin x0 = 3 s, i xn = 2xn−1 + n sin n, ∀ n ≥ 1.
a) Arătat, i că xn > 0, pentru orice n ∈ N.

b) Demonstrat, i că s, irul (yn )n≥2 definit prin yn = n xn este convergent s, i
calculat, i-i limita.

Stelian Corneliu Andronescu s, i Costel Bălcău, Pites, ti

Solut, ia 1. Deoarece −1 ≤ sin n ≤ 1, definind s, irurile (an )n≥0 s, i (bn )n≥0 prin

a0 = b0 = 3, an = 2an−1 − n, bn = 2bn−1 + n, ∀ n ≥ 1,

obt, inem că an ≤ xn ≤ bn , ∀ n ∈ N (induct, ie).


ak ak−1 k n a ak−1  n k
P k P
Avem k = k−1 − k , deci k
− k−1
=− k
. Folosind identitatea
2 2 2 k=0 2 2 k=0 2
n nq n+2 − (n + 1)q n+1 + q an a0
kq k =
P
2
pentru orice q 6= 1, obt, inem n − 0 =
k=0  (q − 1) 2 2
n n+1 1
−4 − n+1 + s, i astfel rezultă că an = 2n + n + 2, pentru orice n ∈ N.
2n+2 2 2
Analog se obt, ine că bn = 5 · 2n − n − 2, pentru orice n ∈ N. Prin urmare

2n + n + 2 ≤ xn ≤ 5 · 2n − n − 2, ∀ n ∈ N,

de unde rezultă imediat că xn > 0 pentru orice n ∈ N s, i, aplicând Criteriul cles, telui,

că lim n xn = 2.
n→∞

Solut, ia 2 (Daniel Văcaru, Pites, ti). a) Demonstrăm că xn > n + 2, ∀ n ∈ N prin


induct, ie. Într-adevăr, x0 = 3 > 2 s, i dacă xk−1 > k + 1, folosind sin k ≥ −1 avem
xk = 2xk−1 + k sin k > 2 (k + 1) − k = k + 2. Deci xn > n + 2 > 0, ∀ n ∈ N.
b) Vom utiliza Criteriul radicalului. Să observăm că lim xn = ∞. Avem
n→∞
xn n sin n n xn n
=2+ s, i −1 ≤ sin n ≤ 1, deci 2 − ≤ ≤2+ , (1).
xn−1 xn−1 xn−1 xn−1 xn−1
Avem xn − xn−1 = xn−1 + n sin n > n + 1 + n sin n > 0, deci xn − xn−1 → ∞.
n 1
Utilizând Lema Stolz-Cesaro, avem lim = lim = 0, deci din (1)
n→∞ xn−1 n→∞ xn − xn−1
xn √
rezultă că lim = 2, prin urmare s, i lim n xn = 2.
n→∞ xn−1 n→∞
 
1
MGO 154. Fie x, y ∈ 0, astfel ı̂ncât 2x + y 3 < 2y + x3 .
2
Demonstrat, i că 3x + y 2 < 3y + x2 s, i reciproc.

Cristinel Mortici, Viforâta


Rezolvarea problemelor din numărul anterior 29
 
1
Solut, ie. Considerăm funct, iile f, g : 0, → R, f (x) = 2x − x3 , g(x) = 3x − x2 .
2
Avem f 0 (x) = 2x ln 2 − 3x2 , f00 (x) = 2x ln2 2 − 6x, f 000 (x) = 2x ln3 2 − 6 ≤
√ 1
2 ln3 2 − 6 < 0, pentru orice x ∈ 0, , deci f 00 este strict descrescătoare. Cum
2

   
2 1 1
00
f (0) = ln 2 > 0 s, i f 00 = 2 ln2 2 − 3 < 0, rezultă că există c ∈ 0,
2   2
1
a.ı̂. f 00 (c) = 0, f 00 (x) > 0 pentru x ∈ [0, c) s, i f 00 (x) < 0 pentru x ∈ c, , deci
  2
1
f 0 este strict crescătoare pe [0, c] s, i strict descrescătoare pe c, . Prin urmare
 2

  
1 1 3
min f 0 (x) ∈ f 0 (0), f 0 . Cum f 0 (0) = ln 2 > 0 s, i f 0 = 2 ln 2 − >
1
x∈[0, 2 ] 2 2 4
 
3 1
1,4 · 0,6 − > 0, rezultă că f 0 (x) > 0 pentru orice x ∈ 0, , deci f este strict
4 2
descrescătoare.
 
1
Pe de altă parte, avem g 0 (x) = 3x ln 3−2x > ln 3−1 > 0 pentru orice x ∈ 0, ,
  2
1
deci g este strict crescătoare. Astfel, pentru orice x, y ∈ 0, avem echivalent, ele
2

2x + y 3 < 2y + x3 ⇔ f (x) < f (y) ⇔ x > y ⇔ g(x) < g(y) ⇔ 3x + y 2 < 3y + x2 .

MGO 155. Fie x, y, z ≥ 0 astfel ı̂ncât xy 2 + yz 2 + zx2 + xyz ≤ 4.


Demonstrat, i că x + y + z ≥ xy + yz + zx.

Leonard Mihai Giugiuc, România s, i Hung Nguyen Viet, Vietnam

Solut, ie. Pentru x + y + z ≤ 3 avem

(x + y + z)2
x+y+z ≥ ≥ xy + yz + zx
3
s, i problema este rezolvată.
Fie acum x + y + z > 3. Presupunem, prin reducere la absurd, că

x + y + z < xy + yz + zx, (1).

Atunci xy + yz + zx > 3. Din Inegalitatea lui Cauchy avem

xy 2 + yz 2 + zx2 (x + y + z) ≥ (xy + yz + zx)2 ,



30 Rezolvarea problemelor din numărul anterior

(xy + yz + zx)2 (xy + yz + zx)2


deci xy 2 + yz 2 + zx2 ≥ . Cum din (1) avem >
x+y+z x+y+z
(xy + yz + zx)2
= xy + yz + zx, rezultă că xy + yz + zx < xy 2 + yz 2 + zx2 , deci
xy + yz + zx
xy + yz + zx + xyz < xy 2 + yz 2 + zx2 + xyz ≤ 4.

Arătăm ı̂n continuare că dacă x, y, z ≥ 0 verifică inegalităt, ile xy + yz + zx > 3


s, i xy + yz + zx + xyz < 4, atunci x + y + z > xy + yz + zx.
Putem presupune, fără a restrânge generalitatea, că z ≥ y ≥ x.
Din xy + yz + zx > 3 s, i yz < 4 avem 1 < yz < 4. Notăm y + z = 2s s, i yz = p2 .
4 − p2
Atunci 1 < p < 2 s, i p ≤ s. Avem x (y + z + yz) < 4 − yz, deci x < . Avem
2s + p2
de demonstrat că xy + yz + zx − (x + y + z) ≤ 0, adică x (2s − 1) −2s + p2 ≤  0.
4−p 2
Fixăm s s, i p s, i considerăm funct, ia f (t) = t (2s − 1) − 2s + p2 , t ∈ 0, .
2s + p2
Deoarece 2s − 1 > 0, avem
4 − p2 (4 − p2 )(2s − 1)
 
h max 2 i f (t) = f 2
= 2
− 2s + p2 .
t∈ 0, 4−p
) 2s + p 2s + p
2s+p2

4 − p2 (4 − p2 )(2s − 1)
 
Astfel xy +yz +zx−(x+y +z) = f (x) < f = −2s+p2 .
2s + p2 2s + p2
4 − p2 (2t − 1)

Fixăm p s, i considerăm funct, ia g (t) = − 2t + p2 , t ∈ [p, ∞).
2t + p2
4 − p2 p2 + 1
 
g 0 (t)
Avem = − 1, pentru orice t ∈ [p, ∞).
2 (2t + p2 )2
4 − p2 p2 + 1
 
2 2
Cum 0 < 4 − p < 3 s, i 2t + p ≥ 2p + p , avem 2 −1 <
(2t + p 2 )2

3 p2 + 1 3 p2 + 1
 
− 1, pentru orice t ∈ [p, ∞) . Arătăm că − 1 < 0, adică
(2p + p2 )2 (2p + p2 )2
3 < p4 + 4p3 + p2 , adevărat. În concluzie avem g 0 (t) < 0 pentru orice  t ∈ [p, ∞),
4 − p 2 (2s − 1)
(2 − p)(2p − 1)
deci max g(t) = g(p) = − 2p + p2 . Astfel −
t∈[p,∞) p 2s + p2
(2 − p) (2p − 1)
2s + p2 = g (s) ≤ g (p) = − 2p + p2 . Arătăm că g(p) < 0, adică
p
(2 − p) (2p − 1)
< p (2 − p), adică 0 < (p − 1)2 , adevărat.
p
Conform inegalităt, ilor demonstrate, rezultă că xy + yz + zx − (x + y + z) <
2

4 − p (2s − 1) (2 − p) (2p − 1)
2
−2s+p2 ≤ −2p+p2 < 0, prin urmare am obt, inut
2s + p p
x + y + z > xy + yz + zx, contradict, ie cu (1). Demonstrat, ia este completă.
Rezolvarea problemelor din numărul anterior 31

Clasa a XII-a

MGO 156. Fie p un număr prim de forma p = 4k + 3, k ∈ N∗ . Demonstrat, i că


 2
ecuat, ia x2 + b
k +b
4 =b9 nu are solut, ii ı̂n Zp .
Stelian Corneliu Andronescu s, i Costel Bălcău, Pites, ti
Solut, ie. Demonstrăm ı̂ntâi că ecuat, ia x2 = −b
1 nu are solut, ii ı̂n Zp .
Într-adevăr, dacă ar exista x ∈ Zp a.ı̂. x2 =n−b 1, atunci x 6= b 0, deci
o
x, b
2x, b \
3x, . . . , (p − 1)x sunt distincte două câte două, x, b
2x, b \
3x, . . . , (p − 1)x =
Z∗p s, i astfel obt, inem, succesiv: x · b 2x · b \
3x · . . . · (p − 1)x = b
1·b
2·b \
3 · . . . · (p − 1);
p−1 4k+2 2
 2k+1 2k+1
x = 1; x
b = 1; x
b = 1; (−1)
b b = 1, fals.
b
Demonstrăm acum că ecuat, ia din enunt, nu are solut, ii prin reducere la absurd.
 2
Să presupunem că există x ∈ Zp a.ı̂. x2 + b k +b4 =b 9. Această ecuat, ie poate
  
fi scrisă sub forma x2 + b k +b1 x2 + b k +b 7 = b 0, deci x2 + b k+b 1 = b 0 sau
x2 + bk +b 7=b 0.
2
Dar ecuat, ia x2 + b
k+b1=b 0 este echivalentă cu x2 + (2k
\ + 1) = b 0 (deoarece
 2
−1
(2k + 1)2 = k(4k + 3) + k + 1), deci cu x · (2k
\ + 1) = −b1, deci nu are solut, ii.

2
Analog, ecuat, ia x2 + b
k +b 0 este echivalentă cu x2 + (2k
7=b \ − 1) = b0 (deoarece
 2
−1
(2k − 1)2 = (k − 2)(4k + 3) + k + 7), deci cu x · (2k\ − 1) = −b1, deci nici ea
nu are solut, ii. Demonstrat, ia este ı̂ncheiată.
MGO 157. Fie n ∈ N, n ≥ 3. Rezolvat, i ı̂n Rn+ sistemul
 5

 x1 + 3 = x22 + 3x3

 5
x2 + 3 = x23 + 3x4 .


 ...
 5
xn + 3 = x21 + 3x2
Mihály Bencze, Bras, ov
n
x5k − x2k − 3xk + 3 = 0, adică
P 
Solut, ie. Adunând ecuat, iile sistemului obt, inem
k=1
n
X
(xk − 1)2 x3k + 2x2k + 3xk + 3 = 0.


k=1

Cum x3k+ 2x2k


+ 3xk + 3 > 0, k = 1, n, rezultă că xk = 1, k = 1, n, care verifică
sistemul dat.
32 Rezolvarea problemelor din numărul anterior

MGO 158. Fie n ∈ N∗ , a ∈ R∗ s, i b > 0. Calculat, i integrala

x2n−1 (x + a)2n−1
Z
dx, x ∈ (0, ∞).
(x + a)4n + bx4n
Daniel Jinga, Pites, ti

Solut, ie. Notând cu I integrala dată, avem


 a 2n−1 Z 1 · 1+ a
 2n−1
x4n−2 1 +
Z
I= x  dx = x2 x dx
a 4n

a 4n
 
x 4n 1+ +b 1+ +b
x x
a 2n 0
 
 a 0  a 2n−1
Z 1+ 1+ Z 1+
1 x x 1 x
=− · dx = − · dx
a
 a 4n 2an

a  2n 2

1+ +b 1+ +b
x x
 a 2n √
1 1 1 + b (x + a)2n
=− · √ · arctg √ x +C =− · arctg √ + C.
2an b b 2abn x2n · b

MGO 159. Fie f : [−1, 1] → R o funct, ie de două ori derivabilă, cu derivata a


doua continuă, astfel ı̂ncât f (−1) = f (1) = 0. Demonstrat, i că are loc inegalitatea
Z 1
1 2
· f 00 (x) dx ≥ max f 2 (x).
6 −1 x∈[−1,1]

Florin Stănescu, Găes, ti

Solut, ie. Concluzia fiind evidentă dacă funct, ia f este identic nulă, presupunem ı̂n
continuare că f nu este identic nulă. Atunci există c ∈ (0, 1) a.ı̂. max f 2 (x) =
x∈[−1,1]
2 2
0 0 0
f (c) > 0. Astfel 0 = f (c) = 2f (c)f (c), deci f (c) = 0.
Z c Z c
00 0
Integrând prin părt, i obt, inem (1 + x) f (x)dx = (1+c)f (c)− f 0 (x)dx =
−1 −1
−f (c), deci aplicând Inegalitatea Cauchy-Buniakowski-Schwarz avem
Z c 2 Z c Z c
00 2 2
2
f (c) = (1 + x) f (x) dx ≤ (1 + x) dx · f 00 (x) dx,
−1 −1 −1

c
3f 2 (c)
Z
2
de unde rezultă că f 00 (x) dx ≥ .
−1 (1 + c)3
1 1
3f 2 (c)
Z Z
00
2
Analog obt, inem că (1 − x) f (x)dx = −f (c) s, i f 00 (x) dx ≥ .
c c (1 − c)3
Rezolvarea problemelor din numărul anterior 33

Adunând cele două inegalităt, i, rezultă că


Z 1  
2 1 1
f 00 (x) dx ≥ 3f 2 (c) + .
−1 (1 + c)3 (1 − c)3
1
2(1 + 3c2 )
Z
1 1 2
Cum + = > 2, rezultă că f 00 (x) dx ≥ 6f 2 (c) =
(1 + c)3 (1 − c)3 (1 − c2 )3 −1
6 · max f 2 (x), adică inegalitatea din enunt, .
x∈[−1,1]

MGO 160. Fie a, b, c, d ≥ 0 astfel ı̂ncât ab + ac + ad + bc + bd + cd = 6.


Demonstrat, i că

2 2 2+ 2
2 2
 √ 
a +b +c +d + · (abc + abd + acd + bcd) ≥ 2 4 + 2 .
2
Când are loc egalitatea?
Leonard Mihai Giugiuc, Drobeta Turnu Severin
Solut, ie. Vom utiliza următoarele două rezultate.
Lema 2 (problema L 355 din RecMat 2/2018). Fie a, b, c, d ∈ R astfel ı̂ncât
a + b + c + d = 4s s, i a2 + b2 + c2 + d2 = 4 3t2 + s2 , cu s, t ≥ 0. Atunci
abc+abd+acd+bcd ≥ (s+t)2 (s−2t), cu egalitate pentru (s + t, s + t, s + t, s − 3t)
s, i permutările sale.
Lema 2 (problema MGO 79 din RMGO 1/2018). Fie α > β > 0 s, i a, b, c, d ≥ 0
astfel ı̂ncât a + b + c + d = 2α + β s, i a2 + b2 + c2 + d2 = 2α2 + β 2 . Atunci
abc + abd + acd + bcd ≥ α2 β, cu egalitate pentru (α, α, β, 0) s, i permutările sale.
Deoarece 3 (a + b + c + d)2 ≥ 8 (ab + ac + ad + bc + bd + cd) = 48, deducem
u2 + 1
că a + b + c + d = 2 · , cu u ≥ 1. Inegalitatea dorită este echivalentă cu
u
2 √
√ 
 2
u +1 2+ 2 
4 − 12 + · (abc + abd + acd + bcd) ≥ 2 4 + 2 .
u 2

 √  u2 + 1 u2 − 1
Cazul 1. u ∈ 1, 2 . Aplicând Lema 1 pentru s = s, i t =
 2u  2u
3
 2 − u2
avem min (abc + abd + acd + bcd) = 2 3u − u , atins ı̂n u, u, u, s, i per-
u
2
√ 
 2
u +1
−12+ 2 + 2 3u − u3 ≥

mutările. Deci este suficient să arătăm că 4
√  √ √ u √  √ 
2 − u (u − 1)2 2 + 2 u2 + 2 + 4 2 u + 2 2 ≥ 0, ine-
 
2 4 + 2 , adică
 √ 
galitate adevărată pentru orice u ∈ 1, 2 . Egalitatea are loc dacă s, i numai dacă
√ √ √ √ 
u = 1 sau u = 2, adică pentru (1, 1, 1, 1) sau 2, 2, 2, 0 s, i permutările sale.
34 Rezolvarea problemelor din numărul anterior
√ !
√ 3+1
Cazul 2. u ∈ 2, √ . Rezolvând sistemul
2
 r  2
u2 +1 2
2· + 4 u u+1 − 18

u2 +1
 
u
 2α + β = 2 · u


 2  α=

2α 2 + β 2 = 4 u2 +1 − 12 avem 3
r
u  2
u2 +1 2
− 2 · 4 u u+1 − 18
 

α>β>0



 2· u
 β=
3
iar inegalitatea de demonstrat devine

√ 2+ 2 √
6· · (abc + abd + acd + bcd) 4+ 2
2
2α + β +2 2 p ≥ · α (α + 2β) .
α (α + 2β) 3

Conform Lemei 2 avem min (abc + abd + acd + bcd) = α2 β, atins ı̂n (α, √ α, β, 0) s, i
√ 2+ 2 2
6· ·α β
permutările. Deci este suficient să arătăm că 2α2 + β 2 + p 2 ≥
√ α (α + 2β)
4+ 2 β
·α (α + 2β) . Notând = x avem x ∈ (0, 1) s, i inegalitatea anterioară devine
3 √ α
√ 2+ 2 √
6· ·x 4+ 2 √
2
x +2+ √ 2 ≥ · (2x + 1). Notând acum 2x + 1 = y avem
2x + 1 3 √
√ 2+ 2
· y2 − 1

√ 
 2
y −1
2 6·
y ∈ 1, 3 , iar inegalitatea devine +2+ 2 ≥
√ 2 2y
4+ 2 √ 2  √ √ √ √ 
· y 2 , adică y − 3 3y 3 + 6 3y 2 − 4 2 − 5 y − 2 6 + 2 3 ≥ 0,

3 √ √
inegalitate adevărată deoarece y > 1 implică 3y 3 > 4 2 − 5 y s, i 6 3y 2 >

√ √
2 6 + 2 3. Mai mult, egalitatea nu poate avea loc.
√  2 2
3+1 u +1
Cazul 3. u ≥ √ . Este suficient să arătăm că 4 − 12 ≥
2 u
2
√  √ 
 2
u +1
2 4 + 2 , inegalitate adevărată deoarece 4 − 12 ≥ 12 > 2 4 + 2 .
u
Mai mult, egalitatea nu poate avea loc nici ı̂n acest caz.
Astfel demonstrat, ia√inegalităt
√ √, ii din
 enunt, este ı̂ncheiată, iar egalitatea are loc
pentru (1, 1, 1, 1) sau 2, 2, 2, 0 s, i permutările sale.
ARTICOLE S, I NOTE MATEMATICE

O extindere a unei probleme din Mathematical


Reflections nr. 6/2021

Titu ZVONARU 1

Punctul de plecare al acestor rânduri este următoarea problemă propusă de


Mihaela Berindeanu ı̂n revista Mathematical Reflections:
Fie ABC un triunghi ascut, itunghic cu ortocentrul H. Notăm cu A1 , B1 , C1
mijloacele laturilor BC, CA, respectiv AB s, i cu A2 , B2 , C2 acele puncte situate
ı̂n interiorul segmentelor HA1 , HB1 , HC1 pentru care
HA2 HB2 HC2
= = =2
A2 A1 B2 B1 C2 C1
Demonstrat, i că dreptele AA2 , BB2 , CC2 sunt concurente.
Prezentăm mai ı̂ntâi o solut, ie pentru un caz put, in mai general, s, i anume atunci
când punctele A2 , B2 , C2 au proprietatea că
HA2 HB2 HC2
= = = k.
A2 A1 B2 B1 C2 C1
Fie O centrul cercului circumscris 4ABC. Dreapta AA2 intersectează dreptele
HO s, i OA1 ı̂n punctele N , respectiv Q.

1
Comănes, ti, t.zvonaru@gmail.com

35
36 Titu ZVONARU

AH HA2
Triunghiurile AHA2 s, i A2 A1 Q sunt asemenea; obt, inem = = k, deci
A1 Q A2 A1
AH (k + 2)AH
A1 Q = . Deoarece AH = 2OA1 , avem OQ = . Aplicând Teorema
k 2k
lui Menelaus ı̂n 4HOA1 cu transversala Q − A2 − N , deducem că
AH
QA1 N O A2 H k NO NO k+2
· · =1⇔ · ·k =1⇒ = .
QO N H A2 A1 (k + 2)AH N H NH 2k
2k
Rezultă că dreptele AA2 , BB2 , CC2 sunt concurente ı̂n punctul N .
Remarcăm că pentru k = 2, punctul N este mijlocul segmentului OH, adică
centrul cercului lui Euler.
Configurat, ia din problemă amintes, te de definit, ia punctului lui Cos, nit, ă:
Dacă O este centrul cercului circumscris triunghiului ABC, iar Oa , Ob , Oc
sunt centrele cercurilor circumscrise triunghiurilor OBC, OCA, respectiv OAB,
atunci dreptele AOa , BOb , COc sunt concurente.
În [1], punctul lui Kosnita este X(54). În partea consacrată punctului X(54),
ı̂n sursa citată se prezintă următoarea construct, ie: fie P, Q două puncte (care
depind de a, b, c). Considerăm A0 = Q pentru 4P BC, B 0 = Q pentru 4P CA,
C 0 = Q pentru 4P AB. Dacă cevianele AA0 , BB 0 , CC 0 sunt concurente, atunci
punctul de concurent, ă se notează cu Kosnita(P, Q) sau K(P, Q). Tot acolo se
ment, ionează relat, ia K(X(4), X(2)) = X(5), ceea ce ı̂nseamnă că dacă H este
ortocentru, iar punctele A2 , B2 , C2 sunt centrele de greutate ale triunghiurilor
HBC, HCA, respectiv HAB, atunci cevianele AA2 , BB2 , CC2 sunt concurente
ı̂n centrul cercului lui Euler (adică exact problema de la care am pornit).
În continuare dorim să demonstrăm următoarea proprietate.
Propozit, ia 1. Fie ABC un triunghi s, i punctele A0 , B 0 , C 0 pe laturile BC, CA,
respectiv AB, astfel ı̂ncât cevianele AA0 , BB 0 , CC 0 sunt concurente ı̂n P . Notăm
cu Ga , Gb , Gc centrele de greutate ale triunghiurilor P BC, P CA, respectiv P AB.
Atunci dreptele AGa , BGb , CGc sunt concurente.
O extindere a unei probleme din Mathematical Reflections nr. 6/2021 37

BA0 CB 0 AC 0
Demonstraţie. Notăm x = , y = , z = (coordonatele baricentrice ale
A0 C B0A C 0B
punctului P ), unde xyz = 1. Fie C1 s, i B1 mijloacele segmentelor CP , respectiv BP .
Notăm {β} = BC1 ∩ AC, {γ} = CB1 ∩ AB, {D} = AGa ∩ BC, {E} = BGb ∩ CA,
{F } = CGc ∩ AB,
Folosind Relat, ia lui Van Aubel s, i Teorema lui Menelaus ı̂n 4ACC 0 cu transver-
sala B − C1 − β, obt, inem

CP 1 CC1 xy + 1 C1 C xy + 1
= +y ⇒ = ⇒ = ;
P C0 x P C0 2x C1 C 0 xy + 2x + 1

BC 0 βA C1 C βA (z + 1)(xy + 2x + 1)
· · 0
=1⇒ = .
BA βC C1 C βC xy + 1
γA (y + 1)(xz + 2z + 1)
Analog, = .
γB y(xz + 1)
Aplicând Teorema lui Ceva ı̂n triunghiul ABC cu cevianele AD, Bβ, Cγ
DB βC γA
deducem că · · = 1, deci
DC βA γB

DB (z + 1)(xy + 2x + 1)y(xz + 1) (xy + 2x + 1)(z + 1)(y + 1)


= =
DC (y + 1)(xz + 2z + 1)(xy + 1) (y + 1)(xz + 2z + 1)(xy + 1)
z(xy + 2x + 1)(z + 1) z(xy + 2x + 1)
= = .
(xz + 2z + 1)(xyz + z) xz + 2z + 1

EC FA
Scriem două relat, ii similare pentru rapoartele s, i :
EA FB
EC x(yz + 2y + 1) F A y(xz + 2z + 1)
= , = .
EA xy + 2x + 1 FB yz + 2y + 1
Cum xyz = 1, aplicând reciproca Teoremei lui Ceva rezultă că dreptele AGa , BGb ,
CGc sunt concurente.

Bibliografie

[1] https://faculty.evansville.edu/ck6/encyclopedia/etc.html
Refinements of Bernoulli’s inequality

Dorin MĂRGHIDANU 1

Bernoulli’s inequality [1], named after its author, the Swiss mathematician Ja-
cob Bernoulli (1654–1705), is one of the most important mathematical inequalities.
It has been known and used since the 17th century, has a very simple expression
and has many applications. For the general case - of real powers, we have the
following statement:
Proposition 1 (Bernoulli’s inequality). If x > −1 and r ∈ R, r ≥ 1, then

(1 + x)r ≥ 1 + rx. (B)

Equality occurs if r = 1 or x = 0.
Remark 1. Inequality (B) also occurs if r ≤ 0. If r ∈ (0, 1), the inequality (B) is
reversed.

Bernoulli’s inequality underlies many famous inequalities, including: means


inequality, Young’s inequality, Hölder’s inequality, Minkowski’s inequality, Radon’s
inequality, and so on. In fact (B) is equivalent to each of the inequalities listed.
For further information on Bernoulli’s inequality, see [2], [3], [4].
Next we will interpose between the members of the inequality (B) certain
expressions - that is to say we will refine the inequality of Bernoulli (sometimes
with slight modifications for the exponent r). Interpolating expressions are also
generated by Bernoulli inequality. It could be said that the obtained refinements
of the Bernoulli inequality are in fact self-refinements!
Proposition 2 (Refinement of Bernoulli’s inequality). If x > −1 and r ∈ R,
r ≥ 2, then
(1 + x)r ≥ (1 + x) [1 + (r − 1)x] ≥ 1 + rx. (B1)
The first inequality becomes equality if r = 2 or x = 0, and the second inequality
becomes equality if x = 0.
Proof. Applying the inequality (B) for the case r − 1, we obtain very simply:
(B)
(1 + x)r = (1 + x) · (1 + x)r−1 ≥ (1 + x) · [1 + (r − 1)x], and then:
(1 + x) [1 + (r − 1)x] = 1 + rx + (r − 1)x2 ≥ 1 + rx. 
1
Profesor dr., Colegiul Nat, ional ,,Al. I. Cuza”, Corabia, d.marghidanu@gmail.com

38
Refinements of Bernoulli’s inequality 39

Proposition 3 (Double refinement of Bernoulli’s inequality). Let x ≥ 0


and r ∈ R, r ≥ 3.
a) We have

(1 + x)r ≥ (1 + 2x)(1 + x)r−2 ≥ (1 + 2x) [1 + (r − 2)x] ≥ 1 + rx. (B3)

The first and the third inequality become equalities if x = 0, and the second
inequality becomes equality if r = 3 or x = 0.
b) We have

(1 + x)r ≥ (1 + x)2 [1 + (r − 2)x] ≥ (1 + 2x) [1 + (r − 2)x] ≥ 1 + rx. (B4)

The first inequality becomes equality if r = 3 or x = 0, and the last two inequalities
become equalities if x = 0.
Proof. a) We apply inequality (B) for the powers 2 and r − 2, successively:
(B) (B)
(1 + x)r = (1 + x)2 · (1 + x)r−2 ≥ (1 + 2x) · (1 + x)r−2 ≥ (1 + 2x) [1 + (r − 2)x] ,

that is the first two inequalities in the statement. For the third inequality, we note
that: (1 + 2x) [1 + (r − 2)x] = 1 + rx + 2(r − 2)x2 ≥ 1 + rx.
b) Similar proof, but with the application of inequality (B) in turn, for the
powers r − 2 and 2. 
A generalization of the last two sentences is given by the following result.

Proposition 4 (Double refinement of Bernoulli’s inequality). Let x ≥ 0


and r, p ∈ R, r ≥ 1 + p, p ≥ 1.
a) We have

(1 + x)r ≥ (1 + px)(1 + x)r−p ≥ (1 + px) [1 + (r − p)x] ≥ 1 + rx. (B5)

The first inequality becomes equality if p = 1 or x = 0, the second inequality becomes


equality if r = p + 1 or x = 0, and the third inequality becomes equality if x = 0.
b) We have

(1 + x)r ≥ (1 + x)p [1 + (r − p)x] ≥ (1 + px) [1 + (r − p)x] ≥ 1 + rx. (B6)

The first inequality becomes equality if r = p + 1 or x = 0, the second inequality


becomes equality if p = 1 or x = 0, and the third inequality becomes equality if
x = 0.
Proof. a) We apply inequality (B) for the powers p and r − p, successively:
(B) (B)
(1 + x)r = (1 + x)p · (1 + x)r−p ≥ (1 + px) · (1 + x)r−p ≥ (1 + px) [1 + (r − p)x] ,
40 Dorin MĂRGHIDANU

so the first two inequalities in the statement. For the third inequality, we note
that: (1 + px) [1 + (r − p)x] = 1 + rx + p(r − p)x2 ≥ 1 + rx.
b) Similar proof, but with the application of inequality (B) for the powers r − p
and p, successively. 
If we decompose the initial power into product by three factors, we can extend
the refinement:

Proposition 5 (Triple refinement of Bernoulli’s inequality). If x ≥ 0 and


m, n, p, r ∈ R≥1 such that r = m + n + p, then

(1 + x)r ≥ (1 + mx)(1 + x)n (1 + x)p ≥ (1 + mx)(1 + nx)(1 + x)p


≥ (1 + mx)(1 + nx)(1 + px) ≥ 1 + rx. (B7)
Proof. We apply inequality (B) for the powers m, n and p, successively:
(B)
(1 + x)r = (1 + x)m (1 + x)n (1 + x)p ≥ (1 + mx)(1 + x)n (1 + x)p
(B) (B)
≥ (1 + mx)(1 + nx)(1 + x)p ≥ (1 + mx)(1 + nx)(1 + px),

that is the first three inequalities in the statement. The fourth inequality results
as follows:

(1 + mx)(1 + nx)(1 + px) = 1 + (m + p + n)x + (mn + np + pm)x2 + mnpx3

= 1 + rx + (mn + np + pm)x2 + mnpx3 ≥ 1 + rx. 


Remark 2. Similar to refinement (B7), five more triple-refinements can be obtained
by permutations, applying the inequality (B) successively for the powers: (m, p, n),
(n, m, p), (n, p, m), (p, m, n), (p, n, m). Obviously, according to the above models,
multiple refinements of Bernoulli inequality can be obtained. The refinements can
also be adapted for the case of r - subunit powers.

References

[1] J. Bernoulli, Positiones arithmeticae de seriebus infinitis earumque summa finita,


(Treatise on Infinite Series), Basel, 1689.

[2] P.S. Bullen, Equivalent Inequalities, arXiv 0809.0641 v2 [math.CA], 2008.

[3] D.S. Mitrinović, J.E. Pečarić, Bernoulli’s Inequality, Rendiconti del Circolo Matematico
di Palermo, vol. 42, no. 3, 1993, pp. 317-337.

[4] D.S. Mitrinović, J.E. Pečarić, A.M. Fink, Classical and New Inequalities in Analysis,
Kluwer Acad. Press., 1993, pp. 65-82.
Some identities and inequalities in triangle

Mihály BENCZE 1

In this paper we present some new inequalities in triangle. We starting with a


well-known problem.

Problem (M 7, Kvant). In any triangle ABC holds the inequality


X a
≥ 3.
b+c−a

Solution. If 2a = y + z, 2b = z + x, 2c = x + y then
 
X a 1Xy+z 1X x y 1X
= = + ≥ 2 = 3.
b+c−a 2 x 2 y x 2
In this paper we give another proof and another relations, inequalities and
generalizations.

Theorem 1. In any triangle ABC we have the relations:

X 1 4R + r
1. = ;
s−a sr
X 1 1
2. = 2;
(s − a)(s − b) r
X 1 (4R + r)2 − 2s2
3. = ;
(s − a)2 s2 r2
X 1 s2 − 2r2 − 8Rr
4. = ;
(s − a)2 (s − b)2 s2 r4
X a 2(2R − r)
5. = ;
s−a r
Y a 4R
6. = ;
s−a r
X a(s − a) 2(2R − r)
7. = ;
(s − b)(s − c) r
1
Profesor dr., Bras, ov, benczemihaly@gmail.com

41
42 Mihály BENCZE
X s−a s2 − 2r2 − 8Rr
8. = ;
(s − b)(s − c) sr2
X (s − a)(s − b) r[s2 + (4R + r)2 ]
9. = ;
c 4sR
X a 2(4R + r)
10. = ;
(s − b)(s − c) sr
X a2 4(R + r)
11. = ;
(s − b)(s − c) R
X a2 4s(R − r)
12. = ;
s−a r
X (s − a)2 s2 − 12Rr
13. = ;
(s − b)(s − c) r2
X ab s2 + r2 − 8Rr
14. = ;
(s − a)(s − b) r2
Xs−a s2 + r2 − 8Rr
15. = ;
a 4Rr
X (s − a)(s − b)2R − r
16. = .
ab 2R
Proof. We proof only the identity 5), the another identities having similar proofs.
X a X a(s − b)(s − c) 1 X
= = 2 a(s − b)(s − c)
s−a (s − a)(s − b)(s − c) sr
1 X 1 X 3 2 2

= a(a − b + c)(a + b − c) = a − (b + c )a + 2abc
4sr2 4sr2
1 Xh X 3 X  X i
= 2 a + 6abc − a a2
4sr2
1   2(2R − r)
= 2
4s(s2 − 3r2 − 6Rr) + 24sRr − 4s(s2 − r2 − 4Rr) = .
4sr r

Corollary 1. In any triangle ABC hold the inequalities:

9 X 1 9R
1. ≤ ≤ ,
2s b+c−a 4sr
that is a new refinement of Euler’s R ≥ 2r Inequality;
X 1 1
2. ≥ 2;
(b + c − a)(a + c − b) R
8R2 − 5r2 X 1 16R2 − 24Rr + 11r2
3. ≤ ≤ ;
4s2 r2 (b + c − a)2 4s2 r2
Some identities and inequalities in triangle 43
 2
8R − 7r X 1 2R − r
4. 2 3
≤ ≤ ;
16s r (b + c − a) (a + c − b)2
2 4sr2
X a
5. ≥ 3,
b+c−a
that is Problem M 7 from Kvant;

6. abc ≥ (a + b − c)(b + c − a)(c + a − b);


X a(b + c − a)
7. ≥ 3;
(a − b + c)(a + b − c)
8R − 7r X b+c−a (2R − r)2
8. ≤ ≤ ;
2sr (a − b + c)(a + b − c) 2sr2
4r(4R2 + 6Rr − r2 ) X (b + c − a)(a − b + c) 4r(5R2 + 3Rr + r2 )
9. ≤ ≤ ;
sR c sR
9 X a 9R
10. ≤ ≤
2s (a − b + c)(a + b − c) 4sr
(another refinement of Euler’s inequality);
3r X a2 3
11. ≤ ≤
R (a − b + c)(a + b − c) 2
(another refinement of Euler’s inequality);
X a2 4s(R − r)
12. ≥ ≥ 2s;
b+c−a R
4R − 5r X (b + c − a)2 4R2 − 8Rr + 3r2
13. ≤ ≤ ;
r (a − b + c)(a + b − c) r2
2R − r X ab R2 − Rr + r2
14. ≤ ≤ ;
r (b + c − a)(a − b + c) r2
2(2R − r) X b + c − a 2(R2 − Rr + r2 )
15. ≤ ≤ ;
R a Rr
6r X (b + c − a)(a − b + c) 2R − r
16. ≤ ≤ .
R ab r
Proof. In Theorem 1 we use the Euler’s Inequality R ≥ 2r and the Gerretsen’s
Inequalities
16Rr − 5r2 ≤ s2 ≤ 4R2 + 4Rr + 3r2 .
We proof only the inequality 5), the another inequalities having similar proofs:
X a 2R − r 4r − r
= ≥ = 3.
b+c−a r r
This is a new proof of Problem M 7, Kvant. 
44 Mihály BENCZE

Theorem 2. In any convex polygon A1 A2 . . . An hold the inequalities:


X a1 n
1. ≥
a2 + a3 + . . . + an n−1
(Nesbitt’s Inequality);
X a1 n
2. ≥
−a1 + a2 + . . . + an n−2
(another generalization of problem M 7, Kvant);
n
X
ak
X a21 k=1
3. ≥ ;
a2 + a3 + . . . + an n−1
n
X
ak
X a21 k=1
4. ≥ .
−a1 + a2 + . . . + an n−2
n
X
Proof. Let S = ak .
k=1

x 2S
1) The function f (x) = is convex because f 00 (x) = > 0. Using
S−x (S − x)3
the Jensen’s Inequality holds the result.
x 4S
2) The function g(x) = is convex because g 00 (x) = > 0.
S − 2x (S − 2x)3
Using the Jensen’s Inequality holds the result.
3) By Cauchy-Schwarz Inequality we get

n
!2 n
!2 n
X X X
ak ak ak
X a21 k=1 k=1 k=1
≥X = n = .
a2 + a3 + . . . + an (a2 + a3 + . . . + an ) X n−1
(n − 1) ak
k=1

4) By Cauchy-Schwarz Inequality we get

n
!2 n
!2 n
X X X
ak ak ak
X a21 k=1 k=1 k=1
≥X = n = .
−a1 + a2 + . . . + an (−a1 + a2 + . . . + an ) X n−2
(n − 2) ak
k=1


Some identities and inequalities in triangle 45

Theorem 3. In any triangle ABC hold the inequalities:

3 X a X a+b
1. + ≥2 ;
2 b+c a + b + 2c
X a Xa+b
2. 3 + ≥ ;
b+c−a c
a + b + c X a2 X (a + b)2
3. + ≥ ;
2 b+c a + b + 2c
X a2 1 X (a + b)2
4. a + b + c + ≥ .
−a + b + c 2 c
x x x2
Proof. The functions f (x) = (for 1.), g(x) = (for 2.), h(x) =
k−x k − 2x k−x
x2
(for 3.), m(x) = (for 4.) where k = a + b + c are convex, and we apply the
k − 2x
Popoviciu’s Inequality. 

References

[1] Octogon Mathematical Magazine (1993-2021).


O dezvoltare a unei probleme din RMM nr.
22/2021

1
Marin CHIRCIU

Această notă pornes, te de la Problema SP. 326 din revista Romanian Ma-
thematical Magazine, propusă de Hoang Le Nhat Tung, Vietnam:
If x, y, z > 0 such that xyz = 1, find the minimum value of expression

xy + yz + zx X x3
+ .
3 (2y 2 − yz + 2z 2 )2

Vom rezolva următoarea dezvoltare a acestei probleme.


Pentru x, y, z > 0 astfel ı̂ncât xyz = 1 s, i n ∈ N, n ≥ 2, să se determine
valoarea minimă a expresiei

xy + yz + zx X x2n−1
P = + .
3n−1 (2y 2 − yz + 2z 2 )n

xy + yz + zx X x2n−1
Solut, ie. Notăm S = s i T = .
(2y 2 − yz + 2z 2 )n
,
3n−1
Pasul 1. Arătăm că
X x2
≥ 1. (1)
2y 2 − yz + 2z 2
Folosind Inegalitatea lui Bergström obt, inem:
X x2 X x4
=
2y 2 − yz + 2z 2 x2 (2y 2 − yz + 2z 2 )
P 2 2 P 4
x + 2 y2z2
P
x
≥P 2 = P 2 2 P ≥ 1,
x (2y 2 − yz + 2z 2 ) 4 y z − xyz x

ultima inegalitate fiind echivalemtă cu


X X X
x4 + xyz x≥2 y2z2,
1
Profesor, Colegiul Nat, ional ,,Zinca Golescu”, Pites, ti, marin.chirciu@yahoo.com

46
O dezvoltare a unei probleme din RMM nr. 22/2021 47

care rezultă folosind Inegalitatea lui Schur s, i Inegalitatea mediilor astfel:


X X X  X X
x4 + xyz x≥ yz y 2 + z 2 ≥ yz · 2yz = 2 y2z2.

Pasul 2. Arătăm că


X x2n−1 1
T = n ≥ n−3 . (2)
2 2
(2y − yz + 2z ) 3 (xy + yz + zx)2
Folosind Inegalitatea lui Hölder obt, inem:
 n P n
x2 x2
X 2y2 −yz+2z 2 2
2y −yz+2z 2 (1) 1 1
T = ≥ ≥ n−2 P ≥ ,
n−2
( yz)2
P
x 3 x 3 x n−3
P
3
 X 2 P
ultima inegalitate fiind echivalentă cu yz ≥ 3 x, care rezultă astfel:

(xy + yz + zx)2 ≥ 3xyz (x + y + z) = 3 (x + y + z) .


Egalitatea are loc pentru x = y = z = 1.
Pasul 3. Arătăm că
xy + yz + zx X x2n−1 4
P = n−1
+ 2 2 n ≥ n−1 . (3)
3 (2y − yz + 2z ) 3
Folosind inegalitatea (2) obt, inem:
xy + yz + zx 1
P =S+T ≥ + ,
3n−1 3 n−3 (xy + yz + zx)2
deci notând t = xy + yz + zx ≥ 3 (conform Inegalităt, ii mediilor ) avem
   
t 1 1 t 3 1 t 9
P ≥ n−1 + n−3 2 = n−2 + ≥ n−2 +
3 3 t 3 3 t2 3 3 t3
  r
1 t t t 9 1 4 t t t 9 1 4 4
= n−2 + + + ≥ n−2 · 4 · · · = n−2 · = n−1 .
3 9 9 9 t3 3 9 9 9 t3 3 3 3

Să trecem acum la determinarea minimului expresiei P din enunt, .


Din inegalitatea (3), cu egalitate pentru x = y = z = 1, rezultă că minimul
4
expresiei P este egal cu n−1 s, i este atins pentru (x, y, z) = (1, 1, 1).
3
Remarcăm că pentru n = 2 se obt, ine rezolvarea Problemei SP. 326.

Bibliografie

[1] Hoang Le Nhat Tung, Problema SP. 326, Romanian Mathematical Magazine, nr.
22/2021.

[2] Marin Chirciu, Inegalităt, i algebrice (2). De la init, iere la performant, ă, Editura Paralela
45, Pites, ti, 2021.
Inegalităt, i pentru determinant, ii matricelor -
cazul general

1
Florin STĂNESCU s, i Carmen VIS, OIU 2

În continuarea articolului din numărul trecut vom prezenta o serie de inegalitat, i
privitoare la determinant, ii matricelor, ı̂n acest număr fiind vizat cazul general.
Vor fi folosite not, iuni uzuale din calculul matriceal, ca de exemplu polinomul
carecteristic s, i valorile proprii ale unei matrice, rangul unei matrice, etc.
Considerăm că n este un număr natural, n ≥ 2.
Aplicat, ia 1. Fie A ∈ Mn (R) o matrice astfel ı̂ncât pentru orice număr natural
nenul m există o matrice simetrică, B ∈ Mn (R), care depinde de m, astfel ı̂ncât
2021B = Am + B 2 . Să se arate că |det A| ≤ 1.
(1)
Solut, ie. Fie λk , k = 1, n valorile proprii ale matricei B, corespunzătoare numărului
(1)
m = 1. Cum B este simetrică, atunci λk ∈ R, (∀) k = 1, n. Mai departe, valorile
(1) 2
 
(1)
proprii ale matricei 2021B − B 2 sunt de forma φk = 2021λk − λk , k = 1, n,
iar din egalitatea din enunt, valorile proprii ale matricei A sunt chiar φk , k = 1, n.
Presupunem prin absurd că există 1 ≤ k ≤ n astfel ı̂ncât |φk | > 1. Cum φm k este
valoare proprie pentru matricea Am , (∀) m > 1, atunci există ϕm ∈ R astfel ı̂ncât
2
φm = 2021ϕ − ϕ 2 , (∀) m > 1. Avem φm = 2021ϕ − ϕ2 ≤ 2021 , (∀) m > 1
k m m k m m
4
∗ m 2p
iar pentru m = 2p, p ∈ N obt, inem că lim φk = lim φk = ∞, contradict, ie, deci
m→∞ p→∞
Qn
|φk | ≤ 1, (∀) k = 1, n ⇒ |det A| = φk ≤ 1.
k=1
Aplicat, ia 2. Fie A ∈ Mn (R) o matrice simetrică cu elementele de pe diagonala
principală egale cu 1 s, i cu suma modulelor elementelor de pe fiecare linie mai mică
sau egală cu 2. Să se arate că det A ≤ 1.
 
x1
Solut, ie. Fie λ o valoare proprie pentru A s, i X =  . . .  un vector propriu. Cum
xn
Pn
A este simetrică, atunci λ ∈ R. Avem AX = λX, deci aij xj = λxi , (∀) i = 1, n.
j=1
1
Profesor, S, coala Gimnazială ,,S, erban Cioculescu”, Găes, ti, florin.florinstanescu@yahoo.com
2
Profesor, Liceul Tehnologic ,,Iordache Golescu”, Găes, ti

48
Inegalităt, i pentru determinant, ii matricelor - cazul general 49
n
P
Întrucât aii = 1, i = 1, n, atunci aij xj = (λ − 1) xi , (∀) i = 1, n. Fie |xk0 | =
j=1
j6=i
 n
P
max |xk | k = 1, n . Astfel, putem scrie |(λ − 1) xk0 | ≤ ak0 j |xk0 | s, i cum
j=1
j6=k0
n
P
ak0 j ≤ 1 rezultă că |λ − 1| ≤ 1, deci λ ∈ (0, 2) . Notând cu λk , k = 1, n valorile
j=1
j6=k0
tr A 1P n √ √
proprii ale matricei A, avem 1 = = λk ≥ n λ1 λ2 . . . λn = n det A.
n n k=1
Aplicat, ia 3. Fie A, B ∈ Mn (R) cu proprietatea că B 2 = In s, i A2 = AB + In .
Să se demonstreze că:
√ !n √ !n
1+ 5 3+ 5
a) det A ≤ ; b) det (A + B) ≤ .
2 2

Solut, ie. a) Avem A (A − B) = In ⇒ A − B = A−1 ⇒ (A − B) A = In ⇒


A2 = BA+In ⇒ AB = BA. Acum, din A2 = AB+In avem A4 = A2 B 2 +2AB+In ,
deci A4 − 3A2 + In = On . Dacă ( λ ∈ C este o valoare proprie a matricei A,
√ √ √ √ )
1+ 5 1+ 5 5−1 5−1
atunci λ4 − 3λ2 + 1 = 0, deci λ ∈ ,− , ,− , deci
2 2 2 2

1+ 5
|λ| ≤ . În final, dacă λ1 , λ2 , . . . , λn sunt valorile proprii ale matricei A,
2 √ !n
n
Q 1+ 5
atunci avem det A ≤ |det A| = |λk | ≤ .
k=1 2

b) Cum valorile proprii ale matricei B sunt −1 sau 1, iar AB = BA, atunci
√ !n √ !n
1+ 5 3+ 5
det (A + B) ≤ |det (A + B)| ≤ 1 + = .
2 2

Aplicat, ia 4. Fie A, B ∈ Mn (R) cu proprietatea că |det (A + zB)| ≤ 1 pentru


orice z ∈ C cu |z| = 1. Să se arate că

(det A) 2 + (det B)2 ≤ 1.

Solut, ie. Fie u ∈ C astfel ı̂ncât un = z, cu |z| = 1, deci s, i |u| = 1. Atunci


z
det (A + zB) = det (A + βC) , unde β = s, i C = uB. Astfel det (A + βC) =
u
2 n
det A + a1 β + a2 β + . . . + det C · β . Dacă ε0 , ε1 , . . . , εn−1 sunt rădăcinile de
ordin n ale unităt, ii, atunci εk0 + εk1 + . . . + εkn−1 = 0, (∀) k = 1, n − 1, de unde
n−1
P n−1
P n−1
P 2 n−1
P n
obt, inem că det (A + εk C) = n det A + a1 εk +a2 εk + . . . +det C εk =
k=0 k=0 k=0 k=0
n−1
P
n (det A + det C). Astfel avem |n (det A + det C)| ≤ |det (A + εk C)| ≤ n, deci
k=0
50 Florin STĂNESCU s, i Carmen VIS, OIU

|det A + det C| ≤ 1, deci |det A + zB| ≤ 1. În final, pentru z = i obt, inem că
|det A + i det B| ≤ 1, adică (det A)2 + (det B)2 ≤ 1.
Aplicat, ia 5. Fie A ∈ Mn (R) astfel ı̂ncât A3 = A + In . Să se arate că det A > 0.

Solut, ie. Ecuat, ia x3 − x − 1 = 0 are o singură rădăcină reală α > 0, prin urmare
f = X 3 − X − 1 = (X − α) X 2 + aX + b , cu a2 < 4b, b > 0. Polinomul minimal


mA al matricei A divide f , deci poate fi de forma X − α sau X 2 + aX + b. Conform


Teoremei lui Frobenius, polinomul minimal mA s, i polinomul caracteristic pA au
t
aceeas, i factori ireductibili, deci pA (X) = (X − α)s X 2 + aX + b , cu s + 2t = n.
În final, (−1)n det A =pA (0) = (−1)s αs bt = (−1)n αs bt , deci det A > 0.
Aplicat, ia 6. Să se demonstreze că dacă o matrice A ∈ Mn (R) are cel put, in
1
n2 − n + 1 elemente mai mici ı̂n modul decât , atunci ea are un minor de ordinul
n
n − 1 mai mare sau egal ı̂n modul decât |det A| .

1
Solut, ie. Din enunt, numărul de elemente mai mari ı̂n modul ca este cel mult
n
n − 1, deci cel put, in o linie a matricei A are toate elementele mai mici ı̂n modul
1
decât . Presupunem că linia k are această proprietate. Atunci |det A| =
n
n n 1 Pn
P P
k+j
(−1) akj kj ≤
δ |akj |·|δkj | ≤ |δkj |. Dacă tot, i minori de ordinul n−1

n j=1

j=1 j=1
1 P n
ar fi mai mici ı̂n modul decât |det A| , atunci |δkj | < |det A| , contradict, ie.
n j=1
Aplicat, ia 7. Fie A ∈ Mn (R) o matrice antisimetrică, adică aij + aji = 0 pentru
orice i, j = 1, n. Să se demonstreze că pentru orice x, y ≥ 0 are loc inegalitatea

det (A + xIn ) · det (A + yIn ) ≥ det (A + xyIn )2 .

Solut, ie. Întrucât A este o matrice antisimetrică, rezultă că valorile proprii λi ,
i = 1, n ale matrice sunt fie nule, fie pur imaginare. Dacă toate valorile proprii
sunt nule, atunci inegalitatea este evident egalitate. Altfel, deoarece elementele
matricei A sunt numere reale, putem considera că avem λ1 = α1 i, λ2 = −α1 i,
. . . , λk−1 = αk−1 i, λk = −αk−1 i, λk+1 = λk+2 = . . . = λn = 0, 1 < k ≤ n, k par,
α1 , . . . , αk ∈ R∗ . Atunci, aplicând Inegalitatea lui Cauchy-Schwarz, avem
k
2 n
√ 2
Y 2 Y
det (A + xyIn ) = αp2 + xy · (xy)
p=1 s=k+1
 k   k 
2
Y n
Y 2
Y n
Y
αp2 + x2 · αp2 + y 2 ·
 
≤ x ·  y
p=1 s=k+1 p=1 s=k+1

= det (A + xIn ) · det (A + yIn ) .


Inegalităt, i pentru determinant, ii matricelor - cazul general 51

Aplicat, ia 8. Fie A, B, C ∈ Mn (R) trei matrice, astfel ı̂ncât

(A − B) (B − C) = A − C.

Să se arate că are loc inegalitatea

det A2 + B 2 + C 2 − AB − BC − CA ≥ 0.


Solut, ie. Din (A − B) (B − C) = A − C obt, inem

(A − B − In ) (B − C − In ) = In ,

de unde rezultă că A − B − In este inversabilă s, i A − B − In = (B − C − In )−1 , deci


s, i (B − C − In ) (A − B − In ) = In . Astfel obt, inem că (B − C) (A − B) = A − C,
deci
(A − B) (B − C) = (B − C) (A − B) .
Efectuând calculele deducem că

AB + BC + CA = BA + CB + AC.

Dacă ε ∈ C\R este o radacină de ordinul trei a unităt, ii, atunci avem

A + εB + ε2 C A + ε2 B + εC
 

= A2 + B 2 + C 2 + ε (BA + CB + AC) + ε2 (AB + BC + CA)


= A2 + B 2 + C 2 + ε + ε2 (AB + BC + CA)


= A2 + B 2 + C 2 − (AB + BC + CA) ,

de unde rezultă că

det A2 + B 2 + C 2 − AB − BC − CA


= det A + εB + ε2 C · det A + ε2 B + εC
 
 
= det A + εB + ε2 C · det A + εB + ε2 C


 2
= det A + εB + ε2 C ≥ 0.

Aplicat, ia 9. Se consideră matricele A, X ∈ Mn (R) astfel ı̂ncât rang X = 1. Să


se arate că
(det(A + X))2 + (det(A − X))2 ≥ 2 · (det A)2 .

Solut, ie. Inegalitatea este evidentă pentru det A = 0.


Fie acum det A 6= 0. Putem scrie

det(A + X) = det A · det(In + A−1 X) s, i det(A − X) = det A · det(In − A−1 X).


52 Florin STĂNESCU s, i Carmen VIS, OIU

Cum rang A−1 X = rang X = 1, atunci polinomul caracteristc corespunzător




matricei A−1 X are forma

pA−1 X (x) = xn − tr A−1 X · xn−1 ,




iar dacă λ1 , λ2 , . . . , λn reprezintă valorile proprii ale matricei A−1 X, atunci avem
−1

λ1 = λ2 = . . . = λn−1 = 0 s, i λn = tr A X . Mai departe, aven

(det(A + X))2 + (det(A − X))2


h 2 2 i
= (det A)2 · det(In + A−1 X) + det(In − A−1 X)
h 2 2 i
= (det A)2 · 1 + tr A−1 X + 1 − tr A−1 X
2
1 + tr A−1 X + 1 − tr A−1 X

2
≥ (det A) ·
2
2
= 2 · (det A) .

Bibliografie

[1] Gh. Andrei, C. Caragea, Gh. Bordea, Algebră pentru concursurile de admitere s, i
olimpiade s, colare, Editura Topaz, Constant, a, 1993.

[2] M. Andronache, R. Gologan, D. Schwarz, D. S, erbănescu, Olimipiada de matematică


2006-2010, Editura Sigma, Bucures, ti, 2010.

[3] A. Chites, , G. Dospinescu, A. Ismail, G. Kreindler, C. Popa, C. Raicu, A. Zahariuc,


Probleme alese de matematică pentru pregătirea Olimpiadei Nat, ionale, Editura Gil,
Zalău, 2010.

[4] G.H. Golub, C.F. Van Loan, Calculul Matriceal, trad. de A. Cipu s, i M. Cipu, Editura
Theta, Bucures, ti, 2005.

[5] F. Stănescu, Probleme de calcul matriceal. Olimpiade, Concursuri s, colare s, i Bacalau-


reat, Editura Cartea Românească Educat, ional, Pites, ti, 2018.

[6] F. Stănescu, Utilitatea unei formule ı̂n rezolvarea unor probleme de calcul matriceal,
RMGO, nr. 1/2019, pg. 23-26.

[7] F. Stănescu, Inegalităt, i pentru determinant, ii matricelor de ordinul 2, RMGO, nr.


1/2020, pg. 37-40.
O formă saturată a Inegalităt, ii AM-GM pentru
triunghi 1

Leonard Mihai GIUGIUC 2

Reamintim câteva forme ale Inegalităt, ii AM − GM pentru trei numere reale


pozitive.
Dacă a, b, c ≥ 0 atunci au loc:
3
a+b+c √

3 a+b+c
≥ abc; ≥ abc;
3 3
s
a2 + b2 + c2 3 a3 + b3 + c3

≥ abc; ≥ abc; etc.
3 3

Rafinarea (s, i chiar saturarea) inegalităt, ilor de mai sus a reprezentat o provocare
continuă de-a lungul anilor, pentru matematicieni.
În continuare vom prezenta o formă saturată a Inegalităt, ii AM − GM pentru
lungimile laturilor unui triunghi.

Teorema 1. Fie a, b s, i c lungimile laturilor BC, CA s, i respectiv AB ale unui


4ABC. Avem
s 3 √
a2 + b2 + c2 2−1
· a3 + b3 + c3 − 3abc .

i) ≥ abc +
3 2

2−1
ii) Dacă β > , atunci inegalitatea
2
s
 2 3
a + b2 + c2
≥ abc + β a3 + b3 + c3 − 3abc

3

nu este ı̂ntotdeauna adevărată.


1
Acest articol a fost comunicat la Simpozionul Judet, ean de Matematică ,,Marinescu-Ghemeci
Octavian”, Edit, ia I, Potcoava, 8 mai 2021.
2
Profesor, Colegiul Nat, ional ,,Traian”, Drobeta Turnu Severin, leonardgiugiuc@yahoo.com

53
54 Leonard Mihai GIUGIUC

Demonstraţie. i) Rescriem inegalitatea sub forma


s
 √ √ 3
a2 + b2 + c2
  
a3 + b3 + c3 + 2 2 − 1 abc ≤ 2 2+1 · . (1)
3

Datorită omogenităt, ii, putem presupune WLOG că

a2 + b2 + c2 = 3.

Deci (1) devine


√ √
a3 + b3 + c3 + (2 2 − 1)abc ≤ 2( 2 + 1). (2)

Utilizăm următorul rezultat.


Lema 1 (Giugiuc, Trăncănău, Pirvuceanu [1]). Fie 0 ≤ a ≤ b ≤ c lungimile
laturilor unui triunghi (chiar degenerat) astfel ı̂ncât a + b + c s, i a2 + b2 + c2 au
valori strict pozitive, fixate. Atunci valoarea maximă a produsului abc se atinge fie
pentru 0 < a = b ≤ c ≤ 2a, fie pentru c = a + b.

Revenim la demonstrat, ia teoremei.


Vom demonstra că (2) este adevărată inclusiv pentru 4ABC degenerat.

Cum a2 + b2 + c2 = 3, atunci ı̂n mod clar 3 ≤ a + b + c ≤ 3.
Prin verificare directă, obt, inem că funct, ia
√ p
f : [0, 1] → [ 3, 3], f (x) = 2x + 3 − 2x2

este o biject, ie strict crescătoare.


Rezolvăm sistemul
 √
 2u + v = 2x + 3 − 2x2
2u2 + v 2 = 3
0≤u≤v


s, i obt, inem u = x, v = 3 − 2x2 .
Să remarcăm că membrul stâng al (2) este o funct, ie crescătoare ı̂n variabila
abc, ı̂n condit, iile ı̂n care avem a + b + c s, i a2 + b2 + c2 fixate.
Deci, conform Lemei 1, este suficient să considerăm două cazuri:
1) 0 < a = b ≤ c ≤ 2a s, i 2) c = a + b.

 
1
În Cazul 1) avem 0 < x ≤ 3− 2x2 ≤ 2x, deci x ∈ √ , 1 .
2
O formă saturată a Inegalităt, ii AM-GM pentru triunghi 55

Astfel este suficient să arătăm că


√ √
 
p 3 p 1
2x3 + 3 − 2x2 + (2 2 − 1)x2 3 − 2x2 ≤ 2( 2 + 1), (∀)x ∈ √ , 1 .
2
Considerăm funct, ia

 
1 p 3 p
g : √ , 1 → R, g(x) = 2x3 + 3 − 2x2 + (2 2 − 1)x2 3 − 2x2 .
2
Avem: √ √
g 0 (x) 4 − 2 2 − (3 − 2 2)x2
 
1
=x− √ , (∀)x ∈ √ ,1 .
6x 3 − 2x2 2
 
0 1
Pentru a studia semnul lui g pe intervalul √ , 1 , este de-ajuns să studiem semnul
2
" √ √ 2 #2 √ !
4 − 2 2 − (3 − 2 2)x 24 − 16 2
lui x2 − √ i.e. semnul lui (1 − x2 ) x2 − √ ,
3 − 2x2 19 − 12 2
s √
1 24 − 16 2
pe acelas, i interval. Cum √ < √ < 1 deducem că g este strict
2 19 − 12 2
" s √ # "s √ #
1 24 − 16 2 24 − 16 2
descrescătoare pe √ , √ s, i strict crescătoare pe √ ,1 .
2 19 − 12 2 19 − 12 2
n   o
1
As, adar, maxh i g(x) ∈ g √
2
, g(1) , ceea ce demonstrează (2).
x∈ √1 ,1
2

În Cazul 2) avem c = a + b, deci a2 + b2 + (a + b)2 = 3.


√ √
Notăm a 2 = x, b 2 = y. Atunci

x2 + y 2 + (x + y)2 = 6

s, i trebuie să arătăm că


√ √ √
x3 + y 3 + (x + y)3 + (2 2 − 1)xy(x + y) ≤ 4 2( 2 + 1).

Fie x + y = 2s s, i xy = p, deci 4s2 − p = 3 s, i trebuie să arătăm că


√ √ √
2 2s3 − ( 2 − 1)sp ≤ 2 + 1.
"√ #
3
Dar 0 ≤ p = 4s2 − 3 ≤ s2 , deci s ∈ , 1 s, i avem de arătat că
2
 √  √  √
4 − 2 2 s3 + 3 2 − 1 s ≤ 2 + 1,

ceea ce este evident adevărat. Deci am demonstrat (1).


56 Leonard Mihai GIUGIUC

2−1
ii) Fie β > , fixat. Presupunem prin reducere la absurd că ine-
s 2
3
a2 + b2 + c2
≥ abc + β a3 + b3 + c3 − 3abc este ı̂ntotdeauna

galitatea
3
s Deciea este adevărată pentru b = c = 1 s, i a = x, (∀)x ∈ (1, 2),
adevărată.
3
x2 + 2
adică ≥ x + β(x3 + 2 − 3x), (∀) ∈ (1, 2), de unde ar rezulta că
3
s
3
√ x2 + 2

≥ lim x + β(x3 + 2 − 3x) = 2 + 4β,
 
2 2 = lim
x→2 3 x→2


2−1
deci β ≤ , contradict, ie. Demonstratia teoremei este ı̂ncheiată.
2

În ı̂ncheiere, propunem spre studiu următoarea problemă.


Se consideră mult, imea

M = (a, b, c) ∈ [0, ∞)3 | b + c ≥ a, c + a ≥ b, a + b ≥ c .




Determinat, i toate valorile β ∈ R pentru care inegalitatea


s
3

 2
a + b2 + c2 h
≥ abc + ( 2 − 1) β (a + b + c)3 +
3
i
+ (1 − 5β) (a + b + c) (ab + bc + ca) + 9(2β − 1)abc

are loc pentru orice (a, b, c) ∈ M .

Bibliografie

[1] http://rmgo.upit.ro/RMGO2/mobile/index.html#p=41

[2] https://www.facebook.com/photo?fbid=10215019109304247&set=g.
400828670030032
TESTE PENTRU EXAMENE

Teste pentru examenul de Evaluare Nat, ională

Costel ANGHEL 1 s, i Florea BADEA 2

Testul 1

SUBIECTUL I
Încercuit, i litera corespunzătoare răspunsului corect.

1. Rezultatul calculului (−14) : 7 + 2,5 este: (5p)

a) 1; b) -2; c) 0,5; d) 1,5.

2. Suma divizorilor naturali ai numărului 9 este: (5p)

a) 10; b) 13; c) 19; d) 5.

3. Dacă 3a · 27b = 243, atunci a + 3b este: (5p)

a) 2; b) 3; c) 4; d) 5.

4. 25% din 50% din 200 este: (5p)

a) 20; b) 30; c) 40; d) 25.

5. Dacă a + b = 8 s, i a − b = 6, a, b ∈ Q, atunci a2 − b2 este: (5p)

a) 47; b) 45; c) 48; d) 49.

6. Fie expresia E(x) = (2x − 3)(2x + 3) + 8, x ∈ R. Numărul pozitiv a, pentru


care E(a) = 3 este: (5p)

a) 2; b) 1,5; c) 3,4; d) 1.

1
Profesor, S, coala Gimnazială ,,Eugen Ionescu”, Slatina, anghelcostel2012@yahoo.com
2
Profesor, S, coala Gimnazială ,,Nicolae Coculescu”, Scornices, ti

57
58 Costel ANGHEL s, i Florea BADEA

SUBIECTUL al II-lea
Încercuit, i litera corespunzătoare răspunsului corect.

1. Pe o dreaptă considerăm punctele A, B, D, C ı̂n această ordine. Dacă D este


mijlocul lui (AC), AB = 4 cm, BC = 8 cm, atunci BD este egal cu: (5p)

a) 1 cm; b) 3 cm; c) 2, 5 cm; d) 2 cm.

2. Complementul suplementului unui unghi de 120◦ , are măsura: (5p)

a) 50◦ ; b) 30◦ ; c) 35◦ ; d) 75◦ .



3. Un triunghi dreptunghic isoscel, cu ipotenuza de 4 2 cm, are perimetrul
egal cu: (5p)
√ √
a) 4(3 + 2) cm; c) 4(2 + 2) cm;
b) 2 dm; d) 0,25 m.

4. Un romb cu diagonala mică de 10 cm s, i un unghi de 120◦ are aria: (5p)


√ √
a) 50 cm2 ; b) 48 cm2 ; c) 50 3 cm2 ; d) 60 2 cm2 .

5. Într-un cerc de centru O s, i rază r = 20 cm se consideră două coarde paralele,


AB = 32 cm s, i CD = 24 cm astfel ı̂ncât punctul O este ı̂n interiorul
patrulaterului ABCD. Distant, a dintre cele două coarde este: (5p)

a) 24 cm; b) 26 cm; c) 28 cm; d) 25 cm.

6. O prismă triunghiulară regulată ABCA0 B 0 C 0 are latura bazei l = 12 cm s, i


^B 0 AB = 30◦ . Volumul acestei prisme este: (5p)
√ √
a) 432 cm3 ; b) 425 3 cm3 ; c) 460 cm3 ; d) 120 3 cm3 .

SUBIECTUL al III-lea
Scriet, i pe foaia de examen rezolvările complete.
p √ √ √
1. Se consideră egalitatea 10 + 2 21 = a 3 + b 7, a, b ∈ R.

a) Scriet, i numărul 10 + 2 21 sub forma (x + y)2 , x, y ∈ R. (2p)
b) Determinat, i perechile (a, b) care verifică egalitatea dată. (3p)

2. Se dă expresia E(x) = (4x − 3)2 − 4(4x − 3) + 4, x ∈ R.


Teste pentru examenul de Evaluare Nat, ională 59

a) Arătat, i că E(x) = 16x2 − 40x + 25, pentru orice x ∈ R. (2p)


b) Arătat, i că există o infinitate de numere naturale n pentru care E(n) se
divide cu 9. (3p)
3. În sistemul de axe ortogonale xOy, se consideră punctele A(3; 4), B(6; 0) s, i
C(3; −4).
a) Reprezentat, i punctele date.
b) Aflat, i perimetrul s, i aria patrulaterului OABC. (3p)
4. Fie ABCD un trapez cu ^A = ^B = 90◦ , AD < BC s, i punctul E ∈ (AB)
astfel ı̂ncât ^AED ≡ ^BEC. Dacă F este mijlocul lui [EC] arătat, i că:
a) 4BEF este isoscel. (2p)
b) BF k DE. (3p)
5. Se dă cubul ABCDA0 B 0 C 0 D0 , AB = 10 cm. S, tiind că M s, i N sunt mijloacele
muchiilor AB, respectiv BC, se cer:
a) distant, a de la D0 la dreapta M N . (2p)
b) distant, a de la A la planul (D0 DM ). (3p)
6. Piramida V ABC are baza triunghiul ABC dreptunghic ı̂n A, AB = 3 dm,
AC = 4 dm s, i ı̂nălt, imea V A = 2,4 dm.
a) Aflat, i măsura unghiului plan corespunzător diedrului determinat de
planele (V BC) s, i (ABC). (2p)
b) Dacă AD ⊥ (V BC), D ∈ (V BC), arătat, i că punctul D este ortocentrul
4V BC. (3p)

Testul 2

SUBIECTUL I
Încercuit, i litera corespunzătoare răspunsului corect.
.
1. Suma cifrelor n pentru care 6n1..3, este: (5p)

a) 12; b) 13; c) 15; d) 16.

2. Cardinalul mult, imii {x ∈ Z||x| ≤ 12} este: (5p)

a) 21; b) 25; c) 24; d) 26.

3. În tabelul următor sunt date rezultatele unui test.


Nota 5 6 7 8 9 10
Nr. elevi 2 4 3 3 3 2
60 Costel ANGHEL s, i Florea BADEA

Numărul de elevi care au luat cel put, in 7 este: (5p)

a) 11; b) 9; c) 8; d) 10.

11
4. Fract, ia este supraunitară dacă s, i numai dacă numărul natural n
3n + 1
apart, ine mult, imii: (5p)

a) {0, 1, 2}; b) {3}; c) {4}; d) {0, 1, 2, 3}.

√ √ √ √
5. Dintre numerele 2 6, 3 3, 2 7, 2 5, cel mai mare este:
√ √ √ √
a) 3 3 b) 2 5; c) 2 7; d) 2 6.

1 1 5
6. Valoarea funct, iei f : R → R, f (x) = x − ı̂n punctul x = , este: (5p)
5 2 2
a) 1; 1 c) 2; d) 0.
b) ;
2

SUBIECTUL al II-lea
Încercuit, i litera corespunzătoare răspunsului corect.

1. Suplementul complementului unui unghi de 20◦ , este: (5p)

a) 70◦ ; b) 100◦ ; c) 109◦ ; d) 110◦ .

2. În figura alăturată, AB k CD s, i


^ACD = 9 · ^BAC.Măsura unghiu-
lui ^BAC este: (5p)

a) 18◦ ; c) 36◦ ;
b) 20◦ ; d) 40◦ .

3. Un trapez cu ı̂nălt, imea de 5 cm s, i linia mijlocie egală cu 8 cm, are aria: (5p)

a) 30 cm2 ; b) 35 cm2 ; c) 38 cm2 ; d) 40 cm2 .


4. Volumul unui cub cu muchia de 5 3 cm este: (5p)
√ √ √
a) 375 cm; b) 375 3 cm; c) 400 3 cm; d) 375 2 cm.
Teste pentru examenul de Evaluare Nat, ională 61

5. În trapezul ABCD, AB k CD (baza mare AB), AD = DC s, i ^ADC = 120◦ .


^BAC are măsura: (5p)

a) 20◦ ; b) 30◦ ; c) 40◦ ; d) 60◦ .

6. Pe cercul C(O; R) se consideră punctele A s, i B astfel ı̂ncât AB = 10 cm s, i


^AOB = 120◦ . Raza cercului, R, este egală cu: (5p)
√ √
a) 10 cm; 10 c) 10 3 cm; 10 2
b) √ cm; d) cm.
3 2

SUBIECTUL al III-lea
Scriet, i pe foaia de examen rezolvările complete.

1. Într-o curte sunt porci s, i găini, ı̂n total 15 capete s, i 40 de picioare.


a) Este posibil ca ı̂n curte să fie 9 găini? Justificat, i. (2p)
b) Câte găini sunt ı̂n curte? (3p)
2. Se dă funct, ia f : R → R, f (x) = 10 − 2x.
a) Calculat, i f (0) · f (1) · f (2) · . . . · f (9). (2p)
b) Aflat, i aria suprafet, ei determinate de axele de coordonate s, i Gf . (3p)
3. Fie E(x) = 9x2 + 30x + 74, x ∈ R.
 
5
a) Calculat, i E − . (2p)
3
b) Aflat, i valoarea minimă a expresiei E(x). (3p)
4. Trapezul ABCD este dreptunghic, ^C = 90◦ , AB este baza mare s, i BD = 20
cm. Dacă punctul E este mijlocul lui AD s, i 4ABD este echilateral, atunci
se cer:
a) Realizat, i un desen corespunzător s, i calculat, i BE. (2p)
b) Aflat, i aria trapezului dat. (3p)
5. Triunghiul ABC este isoscel, AB = AC = 20 cm, BC = 30 cm. Se cer:
a) Aria 4ABC. (2p)
b) Sinusul unghiului ^BAC. (3p)

6. Îm piramida regulată V ABC, cu vârful V , AB = 10 2 cm s, i m(^AV B) =
90◦ . Calculat, i:
a) Distant, a de la punctul A la planul (V BC). (2p)
b) Cotangenta unghiului dintre dreapta V C s, i planul (ABC). (3p)
62 Costel ANGHEL s, i Florea BADEA

Testul 3

SUBIECTUL I
Încercuit, i litera corespunzătoare răspunsului corect.

1. Numărul numerelor de trei cifre divizibile cu 11 este: (5p)

a) 79; b) 80; c) 81; d) 82.

20 15 21 16
√ numerele A = 2 : (−2) , B = (−7) · (−4), C = 2 : (−2) ,
2. Dintre
D = 25 · 6, cel mai mare este: (5p)

a) A; b) B; c) C; d) D.
 
1 1 1
3. Fie x = 0,2+0, (3)+1,1(6) s, i y = + : . Dintre următoarele afirmat, ii,
4 5 6
cea adevărată este: (5p)
y
a) x > 1,8; b) |x − y| = 1; c) x = y; d) < 1.
x

4. Afirmat, ia ,,Cu 38 lei se pot cumpăra 3 pâini de 3,5 lei, 2 ı̂nghet, ate de 5 lei
s, i 2 prăjituri de 9 lei” este: (5p)

a) adevărată; b) falsă.

√ √ √
r
7
5. Dintre numerele m = 62 + 82 , n = − 2 · 8, p = − 32 + 9 s, i q = 2 , cel
9
irat, ional este: (5p)

a) p; b) m; c) n; d) q.

6. S, tiind că numerele 6 s, i x sunt direct proport, ionale cu 15 s, i respectiv 30,


atunci x este: (5p)

a) 10; b) 11,5; c) 12; d) 9.

SUBIECTUL al II-lea
Încercuit, i litera corespunzătoare răspunsului corect.

1. Pe o dreaptă luăm punctele A, B, C


s, i D ca ı̂n figură, astfel ı̂ncât C este
mijlocul lui AD, AD = 20 cm s, i
BD = 12 cm.
Teste pentru examenul de Evaluare Nat, ională 63

Lungimea segmentului AB este: (5p)

a) 10 cm; b) 9 cm; c) 8 cm; d) 7 cm.

2. În patrulaterul convex ABCD, ^A = 100◦ s, i ^D = 60◦ . Suma măsurilor


unghiurilor ^B s, i ^C este: (5p)

a) 180◦ ; b) 200◦ ; c) 80◦ ; d) 90◦ .

3. În figura alăturată ^AOB = 80◦ ,


CO ⊥ OA s, i punctele B, O, D sunt
coliniare. Măsura unghiului ^COD
este: (5p)

a) 180◦ ; c) 160◦ ;
b) 150◦ ; d) 170◦ .

4. În figura alăturată a k b. Cu datele


din figură, x este egal cu: (5p)

a) 50◦ ; c) 80◦ ;
b) 30◦ ; d) 40◦ .

5. Schit, a unui parc este un paralelogram ABCD, AB = 500 m, BC = 600 m


s, i ^BAD = 150◦ . Aria parcului este: (5p)

a) 8 ha; b) 2 ha; c) 15 ha; d) 14 ha.

6. În figura alăturată, AD ⊥ BC,


H este ortocentrul 4ABC. Dacă
m(^C) = 60◦ atunci măsura unghiu-
lui ^BHD este: (5p)

a) 45◦ ; c) 60◦ ;
b) 50◦ ; d) 75◦ .

SUBIECTUL al III-lea
Scriet, i pe foaia de examen rezolvările complete.

1. Media aritmetică a numerelor x s, i y este egală cu 15. Dacă mărim pe x cu 4


s, i mics, orăm pe y cu 2 primul devine de 3 ori mai mare ca al doilea.
64 Costel ANGHEL s, i Florea BADEA

a) Arătat, i că x + y = 30. (2p)


b) Determinat, i numerele x s, i y. (3p)

2. Se dă funct, ia f : R → R, f (x) = (a − 2)x + b, a s, i b rat, ionale.
√ √
a) Determinat, i a, b ∈ Q s, tiind că A( 2; 3 2+4) apart, ine graficului funct, iei
f. (2p)
b) Pentru a = 3 s, i b = 6 calculat, i distant, a de la O, originea sistemului de
axe, la reprezentarea grafică a funct, iei f . (3p)

3. Într-o clasă de elevi, ı̂ntr-o zi numărul celor absent, i reprezintă 10% din
numărul celor prezent, i.

a) Dacă ı̂n clasă sunt 22 de elevi, cât, i sunt absent, i? (2p)


b) Dacă sunt 4 absent, i, cât, i elevi sunt ı̂n total ı̂n acea clasă? (3p)

4. Fie ABCD un pătrat cu AB = 6 cm,


BCE un triunghi isoscel, E ı̂n exte-

riorul pătratului, BE = CE = 3 2
cm. Dacă AE ∩ BC = {P }, se cer:
a) Stabilit, i pozit, ia dreptelor AC s, i
BE. (2p)
b) Ce procent din BC reprezintă
BP ? (3p)

5. În rombul ABCD, ^A < ^B, punctele M s, i N sunt mijloacele laturilor CD,
respectiv AD s, i AB = 1 m.

a) Comparat, i BM s, i BN . (2p)
b) Aflat, i aria patrulaterului DM BN , dacă ^A = 60◦ . (3p)

6. Fie V ABCD o piramidă √ patrulateră regulată cu muchia bazei AB = 10 cm


s, i ı̂nălt, imea V O = 10 cm.

a) Calculat, i aria triunghiului V AC. (2p)


b) Calculat, i sinusul unghiului format de planele (V AC) s, i (V BC). (3p)
Teste pentru examenul de Bacalaureat,
specializarea s, tiint, e ale naturii

Mihai Florea DUMITRESCU 1

Testul 1

SUBIECTUL I (30 de puncte)

1. Se considerǎ numerele complexe z1 = −2, z2 = 2i s, i z3 = 2 + 4i. Arǎtat, i cǎ


z1 − z2
numǎrul este real.
z1 − z3
2. Se considerǎ funct, ia f : R → R, f (x) = x2 + 4x − a, unde a este un numǎr
real. Aflat, i a ∈ Z, astfel ı̂ncât ecuat, ia f (x) = 0 sǎ aibǎ ambele rǎdǎcini reale
s, i de acelas, i semn.

3. Rezolvat, i ı̂n mult, imea numerelor reale ecuat, ia log3 3 x + 1 = 1.

4. Calculat, i probabilitatea ca alegând un numǎr din mult, imea

A = A010 , A110 , A210 , . . . , A10



10 ,

acesta sǎ fie divizibil cu 7.

5. În reperul cartezian xOy se considerǎ punctele A (1, 4), B (−2, 0) s, i C (a, 0)
unde a este un numǎr real. Aflat, i a ∈ R, astfel ı̂ncât AB = 2 · BC.
1
6. În triunghiul ABC se cunoas, te AC = 6, BC = 8 s, i sin A − sin B = . Aflat, i
4
raza cercului circumscris triunghiului ABC .

SUBIECTUL al II-lea (30 de puncte)


 
a + 1 −1
1. Se considerǎ matricele A (a) = , unde a este un numǎr real,
−3 a − 1
   
1 0 0 0
I2 = s, i O2 = .
0 1 0 0
1
Profesor, Liceul ,,S, tefan Diaconescu”, Potcoava, florin14mihai@yahoo.com

65
66 Mihai Florea DUMITRESCU

a) Arǎtat, i cǎ A (1) + A (−1) − 2A (0) = O2 .


b) Aflat, i numărul natural n pentru care are loc egalitatea

det (A (1))t + A (n) = 5.




c) Rezolvat, i ı̂n mult, imea numerelor reale ecuat, ia

det (A (0) − xI2 ) = det (A (1) − xI2 ) .

2. Pe mult, imea numerelor reale se defines, te legea de compozit, ie asociativǎ


1
x ∗ y = xy − (x + y) + 10.
5
1
a) Arǎtat, i cǎ x ∗ y = (x − 5)(y − 5) + 5, pentru orice numere reale x s, i y.
5
b) Gǎsit, i douǎ numere ı̂ntregi a s, i b, astfel ı̂ncât a ∗ b este pǎtrat perfect.
c) Rezolvat, i ı̂n mult, imea numerelor reale ecuat, ia x ∗ x ∗ x = 0.

SUBIECTUL al III-lea (30 de puncte)

1 1
1. Se considerǎ funct, ia f : (0, +∞) → R, f (x) = √ − 2 .
x x
f (x)
a) Calculat, i lim .
x→+1 x−1
b) Aflat, i ecuat, ia asimptotei verticale la graficul funct, iei f .
c) Stabilit, i intervalele de monotonie ale funct, iei f .

2. Se considerǎ funct, ia f : (0, +∞) → R, f (x) = ln2 x + ln x + 1.


Z 2
a) Calculat, i [f (x) − (ln x + 1) ln x] · x dx.
1
Z e
f (x) − 1
b) Calculat, i √ dx.
1 ln x
c) Aflat, i numǎrul real a > 1, astfel ı̂ncât sǎ aibǎ loc egalitatea
Z a
(f (x) − ln x − 1) dx = 2a − 2.
1
Teste pentru examenul de Bacalaureat, specializarea s, tiint, e ale naturii 67

Testul 2

SUBIECTUL I (30 de puncte)

1. Într-o progresie aritmetică (an )n≥1 se s, tie cǎ a1 + a3 = 12 s, i a4 + a6 = 30.


Aflat, i suma a2 + a5 .

2. Determinat, i coordonatele punctelor de intersect, ie a graficelor funct, iilor


f : R → R, f (x) = −x2 + 7 s, i f : R → R, f (x) = x2 − 1.
√ √
3. Rezolvat, i ı̂n mult, imea numerelor reale ecuat, ia x2 + 7 = 1 + x2 .

4. Calculat, i probabilitatea ca alegând o submult, ime cu douǎ elemente din


mult, imea A = {0, 1, 2, 3, 4, 5, 6}, suma elementelor acesteia sǎ fie pǎtrat
perfect.

5. Într-un reper cartezian xOy punctele A(3, 2) s, i B(a, 2) sunt simetrice fat, ǎ
de dreapta x = 1. Aflat, i numǎrul real a.

π 2
6. Se considerǎ triunghiul ABC cu BC = 6, A = , cos B = . Aflat, i AC .
6 2

SUBIECTUL al II-lea (30 de puncte)

 
x 0 x+1
1. Fie matricea A (x) =  0 1 0 , unde x ∈ R.
x−1 0 x

a) Calculat, i det (A (1))2 .


b) Arǎtat, i cǎ matricea A (x) este inversabilǎ pentru orice numǎr real x.
c) Rezolvat, i ı̂n M3 (R) ecuat, ia X · A (1) = A (−1).

2. Se considerǎ inelul (Z12 , +, ·)

a) Arǎtat, i cǎ 2̂2022 + 3̂2022 = 1̂.



2̂x + 5̂y = 1̂
b) Rezolvat, i ı̂n inelul Z12 sistemul de ecuat, ii .
3̂x + 8̂y = 7̂
c) Arătat, i că funct, ia f : U (Z12 ) → U (Z12 ), f (x) = x2 nu este surjectivǎ.

SUBIECTUL al III-lea (30 de puncte)

1. Se consideră funct, ia f : R∗ → R, f (x) = x − ln x2 .

a) Calculat, i lim f (x).


x→+∞
68 Mihai Florea DUMITRESCU

b) Determinat, i ecuat, ia tangentei la graficul funct, iei f ı̂n punctul de abscisǎ


x = 1 situat pe graficul funct, iei.
e
c) Arătat, i că f (x) ≥ 2 ln , oricare ar fi x > 0.
2
x3 1
2. Fie funct, ia f : R → R, f (x) = 2
− 2 .
x +x+1 x +x+1
Z 1
1
a) Arătat, i că f (x) dx = − .
0 2
Z 3
1
b) Calculat, i dx.
2 f (x)
Z a
f (x) + 1 ln 5
c) Calculat, i a > 0, astfel ı̂ncât dx = .
0 x2 + 1 2

Testul 3

SUBIECTUL I (30 de puncte)

3n2 + n
1. Cunoscând suma Sn = a primilor n termeni ai unei progresii
2
aritmetice (an )n≥1 , calculat, i primul termen s, i rat, ia progresiei.

2. Determinat, i numerele reale m, s, tiind că funct, ia f : R → R, f (x) = x + m2


verifică relat, ia (f ◦ f ) (1) = 9.

3. Rezolvat, i ı̂n mult, imea numerelor reale ecuat, ia 3x + 9x = 2 · 27x .

4. Calculat, i probabilitatea ca, alegând un număr din mult, imea

A = {log2 2, log22 2, log23 2, . . . , log210 2} ,

acesta sǎ fie mai mic decât 0,5.

5. În pǎtratul ABCD cu AB = 1 notǎm cu O centrul sǎu. Aflat, i lungimea


−→ −−→
vectorului OA + OB.
π π 
6. S, tiind că sin x = sin , x ∈ , π , calculat, i cos 2x.
3 2

SUBIECTUL al II-lea (30 de puncte)

√ √
1. În reperul cartezian xOy, fie punctele An (n, n) s, i Bn (10 − n, n + 1),
n ∈ N∗ .

a) Determinat, i ecuat, ia dreptei A1 A4 .


Teste pentru examenul de Bacalaureat, specializarea s, tiint, e ale naturii 69

b) Aflat, i numǎrul natural nenul n, astfel ı̂ncât punctele O, An , Bn sunt


coliniare.
c) Aflat, i numǎrul natural nenul n, astfel ı̂ncât aria triunghiului OA1 An sǎ
fie egalǎ cu 1.

2. Pe mult, imea numerelor reale se defines, te legea de compozit, ie asociativǎ

x ∗ y = −xy + 3(x + y) − 6.

a) Calculat, i 0 ∗ (−1).
b) Aflat, i elementul neutru al legii ,,∗”.
c) Calculat, i C22 ∗ C32 ∗ C42 ∗ . . . ∗ C2022
2 .

SUBIECTUL al III-lea (30 de puncte)


x x2 + 4
1. Se consideră funct, ia f : R → R, f (x) = .
x2 + 1
a) Arǎtat, i cǎ f 0 (0) = 2.
b) Aflat, i ecuat, ia asimptotei orizontale spre −∞ la graficul funct, iei f .

2 3
c) Arǎtat, i cǎ |f (x)| ≤ , pentru orice x ∈ R.
3
2. Se consideră funct, ia f : R → R, f (x) = |x| e|x| .
Z 2
f (x)
a) Calculat, i dx.
1 x
Z 1
b) Calculat, i f (x) dx.
−1
Z 1
8
c) Arǎtat, i cǎ f (x2 ) dx ≥ .
0 15
Teste pentru examenul de Bacalaureat,
specializarea matematică-informatică

Costel BĂLCĂU 1

Testul 1

SUBIECTUL I (30 de puncte)

(7 − i)(1 − 7i)
1. Fie numărul complex z = . Calculat, i |z|.
(3 + 4i)2

2. Fie funct, ia f : R → R, f (x) = x2 − x − 1. Determinat, i numerele reale m s, i


n pentru care dreapta de ecuat, ie y = mx + n intersectează graficul funct, iei
f ı̂n punctele de abscise 1 s, i 2.

3. Rezolvat, i ı̂n mult, imea numerelor reale ecuat, ia log2 (1 − x2 ) − 2 log2 x = 3.

4. Calculat, i probabilitatea ca alegând o submult, ime cu 5 elemente a mult, imii


{0, 1, 2, . . . , 8}, aceasta să aibă exact două elemente pare.

5. În reperul cartezian xOy se consideră punctele A(3, 0), B(−2, −1) s, i C(0, 5).
Determinat, i coordonatele simetricului punctului A fat, ă de mijlocul segmen-
tului [BC].

6. Determinat, i x ∈ [0, 2π], s, tiind că sin x − cos x = −1.

SUBIECTUL al II-lea (30 de puncte)



 2x + y + z = a
1. Se consideră sistemul de ecuat, ii ax + 2y = 1 , unde a ∈ R.
x − y + az = 0

a) Determinat, i a ∈ R pentru care sistemul are solut, ie unică.


b) Determinat, i a ∈ R pentru care sistemul este incompatibil.
c) Rezolvat, i sistemul pentru a = 1.
1
Conf. univ. dr., Universitatea din Pites, ti, cbalcau@yahoo.com

70
Teste pentru examenul de Bacalaureat, specializarea matematică-informatică 71

2. Pe mult, imea M = (0, +∞) se defines, te legea de compozit, ie asociativă



x ◦ y = xlg y .

a) Arătat, i că numerele 2021 ◦ 100 s, i 100 ◦ 2022 sunt naturale.


b) Demonstrat, i că legea ,,◦” este comutativă.

c) Se consideră funct, ia f : M → R, f (x) = lg x. Demonstrat, i că
f (x ◦ y ◦ z) = f (x)f (y)f (z), pentru orice x, y, z ∈ M .

SUBIECTUL al III-lea (30 de puncte)


r
4−x
1. Se consideră funct, ia f : (−4, 4] → R, f (x) = .
4+x

f (x) − 3
a) Calculat, i lim .
x→−2 x+2
√  √ 
b) Demonstrat, i că f 2 >f 33
c) Demonstrat, i că funct, ia f are un singur punct de inflexiune.
x−1
2. Se consideră funct, ia f : R → R, f (x) = .
ex
Z 1
a) Calculat, i f (−x) dx.
0
b) Fie F : R → R o primitivă a funct, iei f . Arătat, i că
Z 2
(F (x) + xf (x)) dx = 2F (2).
0
 Z x 
1
c) Calculat, i lim 2
· f (t) dt .
x→1 f (x) 1

Testul 2

SUBIECTUL I (30 de puncte)

1
1. Fie numărul A = 2 log3 15 + − log3 50. Arătat, i că A ∈ N.
log2 3
2. Fie funct, ia f : R → R, f (x) = 3x2 + 2x − 5. Determinat, i a ∈ Z pentru care
(f ◦ f )(a) = −5.
√ √
3. Rezolvat, i ı̂n mult, imea numerelor reale ecuat, ia x + 2 = 2 − x.

4. Determinat, i numărul de elemente ale unei mult, imi, s, tiind că aceasta are
exact 72 de submult, imi ordonate cu două elemente.
72 Costel BĂLCĂU

5. Se consideră triunghiul ABC având centrul de greutate G. Dacă punctul


P este mijlocul segmentului [AG], determinat, i numărul real k astfel ı̂ncât
−−→ −−→ −→
P B + P C = k P A.

6. Determinat, i x ∈ [0, 2π), s, tiind că sin 2x = 2 3 cos2 x.

SUBIECTUL al II-lea (30 de puncte)


 
1 x 2y
1. Pentru orice x, y ∈ R se consideră matricea A(x, y) =  1 y 2x .
1 2 xy

a) Calculat, i det (A(3, 2)).


b) Determinat, i a ∈ R pentru care matricea A(a, a + 1) nu este inversabilă.
c) Demonstrat, i că nu există m, n ∈ Z astfel ı̂ncât det (A(m, n)) = 2021.

2. Pe mult, imea numerelor complexe se defines, te legea de compozit, ie asociativă

z1 ∗ z2 = z1 z2 − (z1 + z2 ) + 2.

a) Calculat, i (1 + i) ∗ (1 − 2i).
b) Determinat, i z ∈ C astfel ı̂ncât z ∗ z = −8.
c) Demonstrat, i că z1 ∗ z2 ∗ z 1 ∗ z 2 ∈ R, pentru orice z1 , z2 ∈ C.

SUBIECTUL al III-lea (30 de puncte)

1. Se consideră funct, ia f : (0, +∞) → R, f (x) = x2 + ln2 x.

a) Arătat, i că f 0 (1) = 2.


b) Determinat, i ecuat, ia asimptotei verticale la graficul funct, iei f .
c) Demonstrat, i că funct, ia f are un unic punct de extrem.
4
2. Se consideră funct, ia f : R → R, f (x) = 1 − .
x2 − 6x + 13
1
(x − 3)2
Z
a) Calculat, i dx.
0 f (x)
Z 4p
b) Calculat, i f (x) dx.
3
Z 2021  n
2
c) Pentru fiecare n ∈ N∗ , fie In = dx. Calculat, i lim In .
2020 f (x) n→+∞
Teste pentru examenul de Bacalaureat, specializarea matematică-informatică 73

Testul 3

SUBIECTUL I (30 de puncte)


√ √ √ √
1. Determinat, i a ∈ R s, tiind că numerele 3 2 − 2 3, a s, i 6 2 + 4 3 sunt
termeni consecutivi ai unei progresii geometrice.

2. Fie funct, ia f : R → R, f (x) = −x2 + 2mx + m, unde m ∈ R. Determinat, i


numerele reale m pentru care graficul funct, iei f intersectează axa Ox ı̂n
două puncte distincte.

3. Rezolvat, i ı̂n mult, imea numerelor reale ecuat, ia 8x+1 = 4x+1 − 2x−1 .

4. Calculat, i probabilitatea ca, alegând un număr din mult, imea numerelor


naturale de trei cifre distincte, acesta să aibă suma cifrelor egală cu 24.

5. Fie ABCD un paralelogram. Dacă punctul M este mijlocul laturii AB, iar
−−→ −−→ −−→
punctul N verifică relat, ia 2N A = N B + 3N D, arătat, i că dreptele M N s, i
BC sunt paralele.

6. Se consideră triunghiul ABC cu AB = AC = 2 3 s, i m(^B) = 30◦ . Calculat, i
lungimea laturii BC.

SUBIECTUL al II-lea (30 de puncte)

 orice x,y ∈ R se consideră matricea M (x + iy) = xI2 + yA, unde


1. Pentru
i 0
A= .
1 −i

a) Arătat, i că det (M (x + iy)) = det (M (y + ix)), pentru orice x, y ∈ R.


b) Demonstrat, i că dacă z1 = x1 +iy1 s, i z2 = x2 +iy2 , unde x1 , y1 , x2 , y2 ∈ R,
atunci M (z1 z2 ) = M (z1 ) · M (z2 ).
c) Determinat, i n ∈ N pentru care (M (1 + i))n = 22020 · I2 .

2. Pe mult, imea numerelor reale se defines, te legea de compozit, ie asociativă


r
3 3
3 x y
x∗y = + x3 + y 3 .
2
r
1
a) Arătat, i că x ∗ y = 3 (x3 + 2)(y 3 + 2) − 2, pentru orice x, y ∈ R.
2
b) Calculat, i simetricul lui −1 ı̂n raport cu legea ..∗”.
c) Rezolvat, i ı̂n mult, imea numerelor reale inecuat, ia

x ∗ x ∗ x ≥ x ∗ x.
74 Costel BĂLCĂU

SUBIECTUL al III-lea (30 de puncte)

1. Se consideră funct, ia f : (−∞, 1) ∪ (2, +∞) → R, f (x) = log2 (x2 − 3x + 2).


1 1
a) Arătat, i că ln 2 · f 0 (x) =+ .
x−1 x−2
b) Determinat, i asimptotele graficului funct, iei f .
c) Calculat, i lim (f (x))log2 |x| .
x→0
x
2. Fie funct, ia f : (0, +∞) → R, f (x) = s, i fie F : (0, +∞) → R o
ex −1
primitivă a funct, iei f .
Z ln 3 x
e f (x)
a) Calculat, i dx.
ln 2 x
b) Arătat, i că F (2020) < F (2021) < F (2022).
c) Arătat, i că F (2020) + F (2022) < 2F (2021).
Cuprins 75

Cuprins

PROBLEME PENTRU CONCURSURI 3

Probleme propuse 3

Rezolvarea problemelor din numărul anterior 11

ARTICOLE S, I NOTE MATEMATICE 35

Titu ZVONARU
O extindere a unei probleme din Mathematical Reflections nr. 6/2021 . . . . . . . 35

Dorin MĂRGHIDANU
Refinements of Bernoulli’s inequality . . . . . . . . . . . . . . . . . . . . . . . . . . 38

Mihály BENCZE
Some identities and inequalities in triangle . . . . . . . . . . . . . . . . . . . . . . 41

Marin CHIRCIU
O dezvoltare a unei probleme din RMM nr. 22/2021 . . . . . . . . . . . . . . . . . 46

Florin STĂNESCU s, i Carmen VIS, OIU


Inegalităt, i pentru determinant, ii matricelor - cazul general . . . . . . . . . . . . . . 48

Leonard Mihai GIUGIUC


O formă saturată a Inegalităt, ii AM-GM pentru triunghi . . . . . . . . . . . . . . 53

TESTE PENTRU EXAMENE 57

Costel ANGHEL s, i Florea BADEA


Teste pentru examenul de Evaluare Nat, ională . . . . . . . . . . . . . . . . . . . . . 57

Mihai Florea DUMITRESCU


Teste pentru examenul de Bacalaureat, specializarea s, tiint, e ale naturii . . . . . . . 65

Costel BĂLCĂU
Teste pentru examenul de Bacalaureat, specializarea matematică-informatică . . . 70
Revistă sponsorizată de SC VISTORIA LUX SRL

S-ar putea să vă placă și